Download as pdf or txt
Download as pdf or txt
You are on page 1of 59

Total Marks : 200

Online Prelims TEST - 5 (SUBJECT WISE)


( InsightsIAS Mock Test Series for UPSC Preliminary Exam 2020 ) Mark Scored : 82

1 With reference to the North Star, consider the following statements:


1. It always remains in the same position in the sky, thus helps to find direction.
2. It is located in the Ursa Major Constellation.

Which of the statements given above is/are correct?


A. 1 only
B. 2 only
C. Both 1 and 2
D. Neither 1 nor 2

Your Answer : C
Correct Answer : A

Answer Justification :

In ancient times, people used to determine directions during the night with the help4of stars. The
03 remains in
North Star indicates the north direction. It is also called the Pole Star. It always
1
the same position in the sky. Hence, statement 1 is correct. 64 4
5
Ursa Minor contains one star with a confirmed planet and has no 87
- Messier objects. The brightest
star in the constellation is Polaris, the North Star (Alphao mUrsae Minoris). Hence, statement 2
is incorrect. il . c
g ma
2@ and third largest constellation in the sky. Its
Ursa Major is the largest northern constellation
2
am
brightest stars form the Big Dipper asterism, one of the most recognizable shapes in the sky, also
p
nu
known as the Plough.
a
d as
s-
2 Consider the following statements:
a
1. Nebular HypothesisD was proposed by Laplace, which considered the planets were formed out of a
m
a associated with a youthful sun.
cloud of material
u p
n to the ‘The Big Bang Theory’, in the beginning, all matter forming the universe existed in
2. According
A
one place in the form of a “tiny ball” with an unimaginably small volume, low temperature and low
density.

Which of the statements given above is/are correct?


A. 1 only
B. 2 only
C. Both 1 and 2
D. Neither 1 nor 2

Your Answer :
Correct Answer : A

Answer Justification :

Nebular Hypothesis was proposed by Pierre Simon de Laplace in 1796, which explained how
the solar system was formed. The hypothesis considered

prelims.insightsonindia.com 1
© Insights Active Learning | All rights reserved - 131815. You may not reproduce, distribute or exploit the contents in any form without
written permission by copyright owner. Copyright infringers may face civil and criminal liability
Total Marks : 200
Online Prelims TEST - 5 (SUBJECT WISE)
( InsightsIAS Mock Test Series for UPSC Preliminary Exam 2020 ) Mark Scored : 82

that the planets were formed out of a cloud of material associated with a youthful sun,
which was slowly rotating. Hence, statement 1 is correct.

The most popular argument regarding the origin of the universe is the Big Bang Theory. It is also
called expanding universe hypothesis. According to the ‘The Big Bang Theory’, in the beginning, all
matter forming the universe existed in one place in the form of a “tiny ball” with an unimaginably
small volume, infinite temperature and infinite density. Hence, statement 2 is incorrect.

3 Which of the following planets is considered as ‘Earth’s-twin’?

A. Mars
B. Mercury
C. Venus
D. Jupiter

4
03
Your Answer : A
1
64
Correct Answer : C
5 4
Answer Justification :
- 87
Venus is the second planet from the Sun, orbiting it every om
.c 224.7 Earth days. It has the longest
i l
a rotates in the opposite direction to most
rotation period of any planet in the Solar System and
gm
other planets. It does not have any natural satellites.
@
2 2
a
Venus is considered as ‘Earth’s-twin’ because m
its size and shape are very much similar to that
u p
of the earth. n a
s
-
Hence, option (c) is correct. da
s
Da
m
u pa
4 Consider the following statements:
An
1. Traditionally, Subsistence Farming utilized low levels of technology and household labour.
2. Commercial grain farming is majorly practiced tropical grasslands of South America, Africa and
Asia.
3. In Mixed farming the land is used for growing crops and rearing livestock.

Which of the statements given above is/are correct?


A. 2 only
B. 1, 2 and 3
C. 1 and 3 only
D. 3 only

Your Answer :
Correct Answer : C

Answer Justification :

Subsistence farming is practiced to meet the needs of the farmer’s family. Traditionally, low levels

prelims.insightsonindia.com 2
© Insights Active Learning | All rights reserved - 131815. You may not reproduce, distribute or exploit the contents in any form without
written permission by copyright owner. Copyright infringers may face civil and criminal liability
Total Marks : 200
Online Prelims TEST - 5 (SUBJECT WISE)
( InsightsIAS Mock Test Series for UPSC Preliminary Exam 2020 ) Mark Scored : 82

of technology and household labour are used to produce on small output. Hence, statement 1 is
correct.

In commercial grain farming crops are grown for commercial purpose. Wheat and maize are
common commercially grown grains. Major areas where commercial grain farming is pracised
are temperate grasslands of North America, Europe and Asia. These areas are sparsely
populated with large farms spreading over hundreds of hectares. Severe winters restrict the
growing season and only a single crop can be grown. Hence, statement 2 is incorrect.

In mixed farming the land is used for growing food and fodder crops and rearing livestock.
It is practiced in Europe, eastern USA, Argentina, southeast Australia, New Zealand and South
Africa. Hence, statement 3 is correct.

5 Consider the following planets:


1. Neptune
2. Jupiter 4
3. Saturn 1 03
4. Uranus 4 64
5
- 87
Which of the above given planets are considered as inner planets?
m
A. 1, 2, and 4 only
i l .co
ma
B. 1, 3 and 4 only
C. 2 only
@ g
D. None 22 m
u pa
Your Answer : D
a n
s
Correct Answer : D
da
a s-
D :
Answer Justification
a m
u
Our Solar psystem consists of eight planets. Out of the eight planets, mercury, venus, earth and
A n
mars are called as the inner planets as they lie between the sun and the belt of asteroids the
other four planets namely Jupiter, Saturn, Uranus and Neptune are called the outer planets.
Alternatively, the first four are called Terrestrial, meaning earth-like as they are made up of rock
and metals, and have relatively high densities. The rest four are called Jovian or Gas Giant planets.

Hence, option (d) is correct.

6 Consider the following statements:


1. The Bering Strait separates the USA and Russia.
2. The Cook Strait connects South Pacific Ocean and the Tasman Sea.

Which of the statements given above is/are correct?


A. 1 only
B. 2 only
C. Both 1 and 2
D. Neither 1 nor 2

prelims.insightsonindia.com 3
© Insights Active Learning | All rights reserved - 131815. You may not reproduce, distribute or exploit the contents in any form without
written permission by copyright owner. Copyright infringers may face civil and criminal liability
Total Marks : 200
Online Prelims TEST - 5 (SUBJECT WISE)
( InsightsIAS Mock Test Series for UPSC Preliminary Exam 2020 ) Mark Scored : 82

Your Answer : C
Correct Answer : C

Answer Justification :

The Bering Strait, located just to the south of the Arctic Circle at the northern edges of the Bering
Sea. This strait separates the United States and Russia by 58 miles (85 km). Hence, statement 1 is
correct.

4
1 03
4 64
5
- 87
.com
a i l
g m
2@
a m2
n up and connects
The Cook Strait is located in New Zealand which acts as a divide between New
s a
Zealand’s North and South Islands, the South Pacific Ocean and the Tasman
a
Sea. Situated next to NewdZealand's capital, Wellington, the straight is widely regarded as
-
sunpredictable.
extremely unsafe and
D a Hence, statement 2 is correct.

p am
u
An

7 The Geological periods Cretaceous, Jurassic and Triassic belong to which of the following Geological
Eras?

prelims.insightsonindia.com 4
© Insights Active Learning | All rights reserved - 131815. You may not reproduce, distribute or exploit the contents in any form without
written permission by copyright owner. Copyright infringers may face civil and criminal liability
Total Marks : 200
Online Prelims TEST - 5 (SUBJECT WISE)
( InsightsIAS Mock Test Series for UPSC Preliminary Exam 2020 ) Mark Scored : 82

A. Cainozoic
B. Mesozoic
C. Palaeozoic
D. None

Your Answer : B
Correct Answer : B

Answer Justification :

The Eras are the second largest interval units into which the Geologic Time is divided after
Eons and represented on the chart. Eras encompass major intervals of Time and are defined based
on the fossil life-forms found in the rock layers, and the Law of Superposition. There are three
Geologic Eras currently identified, namely the Paleozoic Era, the Mesozoic Era, and the
Cenozoic Era.

The Mesozoic Era (248 Million to 65 Million Years Ago) was important for the fossil
0 34remains of the
dinosaurs and other reptiles that lived. However, the Mesozoic Era landscape
6 41was also occupied by
54 flowering plants and
insects, early mammals, plants such as conifers and ferns, fish, and finally
7
-8
early birds. It is further divided into Cretaceous, Jurassic and Triassic Geological periods.
Hence, option (b) is correct. m o
i l .c
a
@ gm
8 Which of the following sources of information are considered as the direct sources of information
about the Earth’s interior? 2
1. Gold mines in South Africa. a m2
2. Volcanic eruption n up
saearth.
3. Meteors that at times reachathe
s -d
Select the correctD a
answer using the code given below:
A. 1 onlyam
up 2 only
B. 1 and
n
C.A3 only
D. 1, 2 and 3

Your Answer : B
Correct Answer : B

Answer Justification :

Most of our knowledge about the interior of the earth is largely based on estimates and inferences.
Yet, a part of the information is obtained through direct observations and analysis of materials.

The most easily available solid earth material is surface rock or the rocks we get from mining areas.
Gold mines in South Africa are as deep as 3 - 4 km. Hence, statement 1 is correct.

Volcanic eruption forms another source of obtaining direct information. As and when the molten
material (magma) is thrown onto the surface of the earth, during volcanic eruption it becomes
available for laboratory analysis. Hence, statement 2 is correct.

prelims.insightsonindia.com 5
© Insights Active Learning | All rights reserved - 131815. You may not reproduce, distribute or exploit the contents in any form without
written permission by copyright owner. Copyright infringers may face civil and criminal liability
Total Marks : 200
Online Prelims TEST - 5 (SUBJECT WISE)
( InsightsIAS Mock Test Series for UPSC Preliminary Exam 2020 ) Mark Scored : 82

Another source of information are the meteors that at times reach the earth. However, it may be
noted that the material that becomes available for analysis from meteors, is not from the interior of
the earth. Thus, it is an indirect source of information. Hence, statement 3 is incorrect.

9 Consider the following statements:


1. Mount Aconcagua is located on the border of Argentina and Paraguay.
2. The Atacama Desert is located in Peru.

Which of the statements given above is/are correct?


A. 1 only
B. 2 only
C. Both 1 and 2
D. Neither 1 nor 2

Your Answer : C
4
Correct Answer : D
1 03
4 64
Answer Justification : 5
- 87
Aconcagua is the highest mountain in both the Southern and m Western Hemispheres. Mount
c o
l. Hence, statement 1 is incorrect.
Aconcagua is located on the border of Argentina and Chile.
a i
m
The driest place on earth is the Atacama Desertgin Chile. It is a plateau that occupies a 600 mile
2
strip of the Pacific coast on the western side2of@Andes Mountains. Hence, statement 2 is
m
incorrect. pa u
n
a sa
- d Waves, consider the following statements:
10 With reference to the Earthquake
s
1. P-waves move fasterathan S-waves.
D
2. P-waves travelmonly through solid medium whereas S-waves travel through both solid and liquid
medium.up
a
n
3. LoveAwaves and Rayleigh waves are body waves whereas P-waves and S-waves are surface waves.

Which of the statements given above is/are correct?


A. 1 only
B. 1 and 3 only
C. 2 and 3 only
D. None

Your Answer : A
Correct Answer : A

Answer Justification :

Earthquake waves are basically of two types — body waves and surface waves. Body waves are
generated due to the release of energy at the focus and move in all directions travelling through the
body of the earth.

prelims.insightsonindia.com 6
© Insights Active Learning | All rights reserved - 131815. You may not reproduce, distribute or exploit the contents in any form without
written permission by copyright owner. Copyright infringers may face civil and criminal liability
Total Marks : 200
Online Prelims TEST - 5 (SUBJECT WISE)
( InsightsIAS Mock Test Series for UPSC Preliminary Exam 2020 ) Mark Scored : 82

There are two types of body waves. They are called P and S-waves. P-waves move faster and are the
first to arrive at the surface. Hence, statement 1 is correct whereas statement 3 is incorrect.

The P-waves are similar to sound waves. They travel through gaseous, liquid and solid
materials. S-waves arrive at the surface with some time lag. These are called secondary waves. An
important fact about S-waves is that they can travel only through solid materials. Hence,
statement 2 is incorrect.

11 Consider the following statements:


1. The size of the parallels of latitude decreases from the Equator towards poles.
2. All meridians are of equal length.

Which of the statements given above is/are correct?


A. 1 only
B. 2 only
4
03
C. Both 1 and 2
1
64
D. Neither 1 nor 2.
5 4
Your Answer : C
- 87
om
Correct Answer : C

i l .c
Answer Justification : a
@ gm
2
All parallel circles from the equator up to the2poles are called parallels
m
of latitudes. Latitudes are measured inadegrees. As we move away from the equator, the size of the
u p
parallels of latitude decreases.
s an Hence, statement 1 is correct.

-
Longitude is the measurementda east or west of the prime meridian. Longitude is measured by
s
Darun around the Earth vertically (up and down) and meet at the North and South
imaginary lines that
m are known as meridians. Each meridian measures one arc degree of longitude.
Poles. Thesealines
u p
Unlike parallels of latitude, all meridians are of equal length. Thus, it was difficult to number
A n
the meridians. Hence, statement 2 is correct.

12 Consider the following statements:


1. Gobi desert is located in China and Mongolia.
2. The Darvaza gas crater is located in the Kyzyl Kum desert.

Which of the statements given above is/are correct?


A. 1 only
B. 2 only
C. Both 1 and 2
D. Neither 1 nor 2.

Your Answer :
Correct Answer : A

Answer Justification :

prelims.insightsonindia.com 7
© Insights Active Learning | All rights reserved - 131815. You may not reproduce, distribute or exploit the contents in any form without
written permission by copyright owner. Copyright infringers may face civil and criminal liability
Total Marks : 200
Online Prelims TEST - 5 (SUBJECT WISE)
( InsightsIAS Mock Test Series for UPSC Preliminary Exam 2020 ) Mark Scored : 82

The Gobi Desert is the 2nd largest desert in Asia after Arabian Desert. It extends from northern
China into Mongolia. Hence, statement 1 is correct.

The Kara Kum Desert covers 135,000 square miles, nearly 70 percent of Turkmenistan's land.
Because of the desert's location along the Caspian Sea, the weather in Karakum is milder than
many Asian deserts. The Darvaza gas crater, also called the "Door to Hell" or the "Gates of Hell"
by locals, a crater of natural gas that has been burning since 1971, is located in the Karakum
Desert. Hence, statement 2 is incorrect.

The Kyzylkum desert lies in the doab sandwiched by two rivers; the Syr Darya and the Amu
Darya, in a section of the world that is historically called Sogdiana or Transoxania. As it stands
today, it spans approximately 115,000 square miles lying in Uzbekistan, Kazakhstan, and
Turkmenistan.

13 Which of the following activities is/are considered primary activities?


1. Agriculture 4
2. Forestry 1 03
3. Construction Industry 4 64
5
- 87
Select the correct answer using the code given below.
A. 1 and 3 only . c om
B. 1 and 2 only a il
C. 2 only
@ gm
D. 1, 2 and 3 22 m
u pa
Your Answer : B
a n
s
Correct Answer : B
da
a s-
D :
Answer Justification
m
pa
Primaryuactivities are directly dependent on environment as these refer to utilisation of
A n
earth’s resources such as land, water, vegetation, building materials and minerals. It, thus includes,
hunting and gathering, pastoral activities, fishing, forestry, agriculture, and mining and
quarrying. Hence, both statement 1 and statement 2 are correct.

Secondary activities add value to natural resources by transforming raw materials into
valuable products. Secondary activities, therefore, are concerned with manufacturing,
processing and construction (infrastructure) industries. Hence, statement 3 is incorrect.

14 Consider the following statements:


1. Kalgoorlie and Coolgardie areas of Western Australia have the large deposits of high grade
anthracite coal.
2. Australia is a major producer of bauxite in the world.
3. Lake Maracaibo is known for petroleum production.

Which of the statements given above is/are correct?


A. 1 and 2 only

prelims.insightsonindia.com 8
© Insights Active Learning | All rights reserved - 131815. You may not reproduce, distribute or exploit the contents in any form without
written permission by copyright owner. Copyright infringers may face civil and criminal liability
Total Marks : 200
Online Prelims TEST - 5 (SUBJECT WISE)
( InsightsIAS Mock Test Series for UPSC Preliminary Exam 2020 ) Mark Scored : 82

B. 2 and 3 only
C. 1, 2 and 3
D. 1 only

Your Answer :
Correct Answer : B

Answer Justification :

Kalgoorlie and Coolgardie areas of Western Australia have the largest deposits of gold. The
oldest rocks in the world are in Western Australia. They date from 4,300 million years ago,
only 300 million years after the earth was formed. Hence, statement 1 is incorrect.

Australia is the largest producer of bauxite in the world. Hence, statement 2 is correct.

Lake Maracaibo is located in Venezuela and known for major petroleum production of Venezuela.
Hence, statement 3 is correct. 34 0
6 41
4
15 Consider the following statements:
8 75
1. Richter scale is used for measuring the intensity of Earthquake. -
c om
2. Mercalli scale is used for measuring the magnitude of Earthquake.
.
3. The range of intensity scale is from 1-10. ail
@ gm
2
Which of the statements given above is/are2correct?
A. 1 and 2 only
pam
B. 3 only n u
C. 1, 2 and 3 a sa
D. None
s -d
Da
Your Answera:m D
Correct u p
Answer : D
An
Answer Justification :

The earthquake events are scaled either according to the magnitude or intensity of the shock. The
magnitude scale is known as the Richter scale. The magnitude relates to the energy
released during the quake. The magnitude is expressed in numbers, 0-10. Hence statement 1 is
incorrect.

The intensity scale is named after Mercalli, an Italian seismologist. The intensity scale takes
into account the visible damage caused by the event. The range of intensity scale is from 1-12.
Hence, statement 2 and statement 3 are incorrect.

16 Which of the following evidences supported the continental drift?


1. Jig-Saw-Fit of the shorelines of Africa and South America.
2. Tillite deposits

prelims.insightsonindia.com 9
© Insights Active Learning | All rights reserved - 131815. You may not reproduce, distribute or exploit the contents in any form without
written permission by copyright owner. Copyright infringers may face civil and criminal liability
Total Marks : 200
Online Prelims TEST - 5 (SUBJECT WISE)
( InsightsIAS Mock Test Series for UPSC Preliminary Exam 2020 ) Mark Scored : 82

3. Placer deposits

Select the correct answer using the code given below:


A. 1 and 2 only
B. 2 and 3 only
C. 1 and 3 only
D. 1, 2 and 3

Your Answer : D
Correct Answer : D

Answer Justification :

According to Wegener, all the continents formed a single continental mass in the beginning and
later continents drifted to form the present day map.

Some of the evidences that supported continental drift were:


0 34
41
The shorelines of Africa and South America facing each other have4a6remarkable and
unmistakable match. The match was tried at 1,000-fathom line instead 8 75 of the present shoreline.
-drifted. Hence, statement 1 is
Thus, theses continents were together in the beginning and later m
correct.
i l .co
g ma of glaciers. The Gondawana system of
Tillite is the sedimentary rock formed out of deposits
@
2counterparts
sediments from India is known to have its
m 2 in six different landmasses of the
Southern Hemisphere.
u pa
n
Overall resemblance of the a sa
Gondawana-type sediments clearly demonstrates that these landmasses
d
had remarkably similar -histories. The glacial tillite provides unambiguous evidence of
s
palaeoclimates andDa also of drifting of continents. Hence, statement 2 is correct.
p amof rich placer deposits of gold in the Ghana coast and the absolute absence of
u
The occurrence
Anrock in the region is an amazing fact. The gold bearing veins are in Brazil and it is
source
obvious that the gold deposits of the Ghana are derived from the Brazil plateau when the two
continents lay side by side. Hence, statement 3 is correct.

17 Consider the following statements:


1. Eritrea and Djibouti share the border with Red Sea
2. River Euphrates drains into Gulf of Oman.

Which of the statements given above is/are correct?


A. 1 only
B. 2 only
C. Both 1 and 2
D. Neither 1 nor 2.

Your Answer :
Correct Answer : A

prelims.insightsonindia.com 10
© Insights Active Learning | All rights reserved - 131815. You may not reproduce, distribute or exploit the contents in any form without
written permission by copyright owner. Copyright infringers may face civil and criminal liability
Total Marks : 200
Online Prelims TEST - 5 (SUBJECT WISE)
( InsightsIAS Mock Test Series for UPSC Preliminary Exam 2020 ) Mark Scored : 82

Answer Justification :

There are six countries (Saudi Arabia, Yemen, Egypt, Sudan, Eritrea, and
Djibouti) bordering the Red Sea. Hence, statement 1 is correct.

4
1 03
4 64
5
- 87
c om
a il.
The Euphrates is the longest and one of the most historically important rivers of Western

@ gm
Asia. Together with the Tigris, it is one of the two defining rivers of Mesopotamia. Originating
in eastern Turkey, the Euphrates flows through
2 2 Syria and Iraq to join the Tigris in the Shatt al-
Arab, which empties into the Persian mGulf. Hence, statement 2 is incorrect.
u pa
a n
18 Which among the following d as
statements best describes the term ‘Black Gold’:
s -
Da
A. Coal and its derivatives
m
aDiamond
p
B. Green
u
An
C. Residual that remain after Gold extraction
D. Petroleum and its derivatives

Your Answer : A
Correct Answer : D

Answer Justification :

Petroleum is found between the layers of rocks and is drilled from oil fields located in off-shore and
coastal areas. This is then sent to refineries which process the crude oil and produce a variety of
products like diesel, petrol, kerosene, wax, plastics and lubricants. Petroleum and its
derivatives are called Black Gold as they are very valuable.

Hence, option (d) is correct.

19 Consider the following statements:


1. Oceanic crust is thinner as compared to the continental crust.
prelims.insightsonindia.com 11
© Insights Active Learning | All rights reserved - 131815. You may not reproduce, distribute or exploit the contents in any form without
written permission by copyright owner. Copyright infringers may face civil and criminal liability
Total Marks : 200
Online Prelims TEST - 5 (SUBJECT WISE)
( InsightsIAS Mock Test Series for UPSC Preliminary Exam 2020 ) Mark Scored : 82

2. The lower portion of the mantle is called asthenosphere.


3. The crust and the uppermost part of the mantle are called lithosphere.

Which of the statements given above is/are correct?


A. 1 and 3 only
B. 2 and 3 only
C. 1, 2 and 3
D. 1 only

Your Answer : A
Correct Answer : A

Answer Justification :

The Crust is the outermost solid part of the earth. It is brittle in nature. The thickness of the
crust varies under the oceanic and continental areas. Oceanic crust is thinner as compared to
the continental crust. The mean thickness of oceanic crust is 5 km whereas that0 3of4 the continental
is around 30 km. Hence, statement 1 is correct. 41 6
4
The upper portion of the mantle is called asthenosphere. It is 8 75
- considered to be extending upto

o m
400 km. It is the main source of magma that finds its way to the surface during volcanic eruptions.
Hence, statement 2 is incorrect. l. c i
m a
@g are called lithosphere. Its thickness ranges
The crust and the uppermost part of the mantle
2
2
from 10-200 km. Hence, statement 3 is correct.
m
u pa
n
a sa
20 With reference to the ‘Mass Movement’, consider the following statements:
d
- rock debris down the slopes under the direct influence of gravity.
1. They transfer the mass of
a s
D
2. Weathering is a pre-requisite for mass movement.
m can be either slow or rapid.
pa
3. Mass movements
u
Anof the statements given above is/are correct?
Which
A. 1 and 3 only
B. 2 only
C. 1, 2 and 3
D. 3 only

Your Answer :
Correct Answer : A

Answer Justification :

Mass movements transfer the mass of rock debris down the slopes under the direct influence
of gravity. That means, air, water or ice do not carry debris with them from place to place but on
the other hand the debris may carry with it air, water or ice. The movements of mass may range
from slow to rapid. Hence, both statement 1 and statement 3 are correct.

Gravity exerts its force on all matter, both bedrock and the products of weathering. So,

prelims.insightsonindia.com 12
© Insights Active Learning | All rights reserved - 131815. You may not reproduce, distribute or exploit the contents in any form without
written permission by copyright owner. Copyright infringers may face civil and criminal liability
Total Marks : 200
Online Prelims TEST - 5 (SUBJECT WISE)
( InsightsIAS Mock Test Series for UPSC Preliminary Exam 2020 ) Mark Scored : 82

weathering is not a pre-requisite for mass movement though it aids mass movements. Hence,
statement 2 is incorrect.

21 Consider the following statements:


1. Thickness of the troposphere is greatest at the poles.
2. Radio waves transmitted from the earth are reflected back to the earth by the ionosphere.
3. The mesosphere contains the Ozone layer.

Which of the statements given above is/are correct?


A. 1 and 2 only
B. 1 and 3 only
C. 2 and 3 only
D. 2 only

Your Answer : D
4
Correct Answer : D
1 03
4 64
Answer Justification : 5
- 87
The column of atmosphere is divided into five different layers
o mdepending upon the temperature
l. c
condition. They are: troposphere, stratosphere, mesosphere, thermosphere and exosphere.
i
m a
@ g
The troposphere is the lowermost layer of the atmosphere. Its average height is 13 km and extends
roughly to a height of 8 km near the poles 2
2 and about 18 km at the equator. Thickness of the
m
a because heat is transported to great heights by
troposphere is greatest at the equator
strong convectional currents.n up statement 1 is incorrect.
Hence,
s a
da
- above the tropopause and extends up to a height of 50 km. One
The stratosphere is found
a s
D of the stratosphere is that it contains the ozone layer. Hence, statement
important feature
a m
3 is incorrect.
nup
The A
ionosphere is located between 80 and 400 km above the mesopause. It contains
electrically charged particles known as ions, and hence, it is known as ionosphere. Radio waves
transmitted from the earth are reflected back to the earth by this layer. Hence, statement 2
is correct.

22 The famous Mount Olympus is located in which of the following Countries?

A. Spain
B. Germany
C. Italy
D. Greece

Your Answer : D
Correct Answer : D

Answer Justification :

prelims.insightsonindia.com 13
© Insights Active Learning | All rights reserved - 131815. You may not reproduce, distribute or exploit the contents in any form without
written permission by copyright owner. Copyright infringers may face civil and criminal liability
Total Marks : 200
Online Prelims TEST - 5 (SUBJECT WISE)
( InsightsIAS Mock Test Series for UPSC Preliminary Exam 2020 ) Mark Scored : 82

Mount Olympus is the highest mountain in Greece. Olympus is notable in Greek mythology as the
home of the Greek gods, on Mytikas peak. It is also noted for its exceptional biodiversity and
rich flora. It has been a National Park, the first in Greece, since 1938. It is also a World
Biosphere Reserve.

23 Which of following factors is/are affecting the distribution of population?


1. Soil
2. Topography
3. Climate

Select the correct answer using the code given below:


A. 1 and 2 only
B. 2 and 3 only
C. 3 only
D. 1, 2 and 3
4
1 03
64
Your Answer : D
Correct Answer : D 5 4
- 87
Answer Justification :
.c om
i l
Distribution of population is affected by geographical,asocio-cultural , economic and political
factors. @ gm
2 2
Fertile soils provide suitable land for
p amagriculture. Fertile plains such as Ganga and
Brahmaputra in India, Hwang-He, u
n Chang Jiang in China and the Nile in Egypt are densely
s a
populated. Thus, soil affectsathe distribution of population. Hence, statement 1 is correct.
s -d
Da to live on plains rather than mountains and plateaus because these
People always prefer
am for farming, manufacturing and service activities. Thus, topography affects the
areas are suitable
p
u of population. Hence, statement 2 is correct.
distribution
An
People usually avoid extreme climates that are very hot or very cold like Sahara desert, polar
regions of Russia, Canada and Antarctica. Thus, climate affects the distribution of population.
Hence, statement 3 is correct.

24 With reference to the shield volcanoes, consider the following statements:


1. The shield volcanoes are the largest of all the volcanoes on the earth by area.
2. The shield volcanoes are very steep in nature.
3. The Hawaiian volcanoes are example for the shield volcanoes.

Which of the statements given above is/are correct?


A. 1 and 3 only
B. 1, 2 and 3
C. 3 only
D. 1 only

prelims.insightsonindia.com 14
© Insights Active Learning | All rights reserved - 131815. You may not reproduce, distribute or exploit the contents in any form without
written permission by copyright owner. Copyright infringers may face civil and criminal liability
Total Marks : 200
Online Prelims TEST - 5 (SUBJECT WISE)
( InsightsIAS Mock Test Series for UPSC Preliminary Exam 2020 ) Mark Scored : 82

Your Answer :
Correct Answer : C

Answer Justification :

Barring the basalt flows, the shield volcanoes are the largest of all the volcanoes on the earth.
Thus, shield volcanoes are second largest of all the volcanoes on the earth. Hence, statement 1 is
incorrect.

The Shield volcanoes are mostly made up of basalt, a type of lava that is very fluid when
erupted. For this reason, these volcanoes are not steep. Hence, statement 2 is incorrect.

The Hawaiian volcanoes are the most famous examples for shield volcanoes. Hence,
statement 3 is correct.

3 4
25 What is/are the reasons that can be attributed to the frequent landslides and debris avalanches in
Himalayan region? 0
1. Himalayas are tectonically stable. 6 41
2. They are mostly made up of sedimentary rocks. 7 54
3. Steeper slopes -8
m
i l . co
Select the correct answer using the code given below:a
A. 1 only
@ gm
B. 2 only
2 2
m
pa
C. 1, 2 and 3
D. 2 and 3 only
n u
a sa
Your Answer : D
s -d
Correct Answer : D a
D
m
Answer u pa
Justification :
An
The reasons that can be attributed to the frequent landslides and debris avalanches in Himalayan
region are

(i) The Himalayas are tectonically active. Thus, tectonically unstable. Hence, statement 1 is
incorrect.

(ii) They are mostly made up of sedimentary rocks and unconsolidated


and semi-consolidated deposits. Hence, statement 2 is correct.

(iii) The slopes are very steep. Hence, statement 3 is correct

26 Consider the following fold mountains:


1. Aravali Ranges
2. The Appalachians
3. The Rockies

prelims.insightsonindia.com 15
© Insights Active Learning | All rights reserved - 131815. You may not reproduce, distribute or exploit the contents in any form without
written permission by copyright owner. Copyright infringers may face civil and criminal liability
Total Marks : 200
Online Prelims TEST - 5 (SUBJECT WISE)
( InsightsIAS Mock Test Series for UPSC Preliminary Exam 2020 ) Mark Scored : 82

4. The Andes

Which of the above given mountains is/are young fold mountains?


A. 2 and 4 only
B. 3 and 4 only
C. 1, 3 and 4 only
D. 1, 2, 3 and 4

Your Answer : B
Correct Answer : B

Answer Justification :

Old Fold Mountains are those fold mountains which had their origin before the Tertiary period.
Prominent examples include Aravali Ranges, Ural Mountains and The Appalachians.

4
Young Fold Mountains are those fold mountains which had their origin after the3Tertiary period.
1 0
Prominent examples include Alps, Rockies, Andes and Himalayas. 4 6
4
75
-8
Hence, option (b) is correct.

o m
27 Consider the following statements: a il.c
g mGulf
1. The Strait of Hormuz is a strait between the Persian and the Gulf of Oman.
2. Dardanelles Strait connects the Black Sea 2
@
to2the Sea of Marmara.
amSea to the Sea of Marmara.
3. The Bosporus Strait connects the Aegean
p
u
an
as above is/are correct?
Which of the statements given
d
A. 1 and 3 only
s -
B. 2 and 3 onlyD a
p am
C. 1, 2 and 3
u
D. 1 only
An
Your Answer : A
Correct Answer : D

Answer Justification :

The Strait of Hormuz is a strait between the Persian Gulf and the Gulf of Oman. It provides the
only sea passage from the Persian Gulf to the open ocean. Hence, statement 1 is correct.

The Dardanelles Strait, a vital transportation bridge between the Black Sea and Mediterranean
Sea, is a narrow channel of water that connects the Aegean Sea to the Sea of Marmara.

It separates Asian Turkey from European Turkey thus it also separates the two continents. Hence,
statement 2 is incorrect.

The Bosporus Strait connects the Black Sea to the Sea of Marmara. It also separates Asian
Turkey from European Turkey, thus it separates the two continents. Hence, statement 3 is

prelims.insightsonindia.com 16
© Insights Active Learning | All rights reserved - 131815. You may not reproduce, distribute or exploit the contents in any form without
written permission by copyright owner. Copyright infringers may face civil and criminal liability
Total Marks : 200
Online Prelims TEST - 5 (SUBJECT WISE)
( InsightsIAS Mock Test Series for UPSC Preliminary Exam 2020 ) Mark Scored : 82

incorrect

28 Consider the following statements:


1. Birth rate is measured as the number of live births per 100 people.
2. Natural Population growth rate of a country is measured as difference between the birth rate and
the death rate of a country.

Which of the statements given above is/are correct?


A. 1 only
B. 2 only
C. Both 1 and 2
D. Neither 1 nor 2

Your Answer : D
Correct Answer : B
4
1 03
Answer Justification :
64
54
87 a specific time. Births are
The population change refers to change in the number of people during
-
usually measured using the birth rate i.e. the number of livembirths per 1,000 people. Hence,
c o
statement 1 is incorrect. l. a i
m
@g change. The difference between the birth
Births and deaths are the natural causes of population
2
2 the natural growth rate. Hence, statement 2 is
rate and the death rate of a country is called
m
correct. pa u
n
a sa
- d volcanic landforms, consider the following statements:
29 With reference to the intrusive
s
Da portion of magma chambers.
1. Batholiths are the cooled
2. Dykes are them near horizontal bodies of the intrusive igneous rocks.
p a
u are wavy mass of intrusive rocks.
3. Phacoliths
An
Which of the statements given above is/are correct?
A. 1 and 3 only
B. 2 and 3 only
C. 1, 2 and 3
D. 1 and 2 only

Your Answer :
Correct Answer : A

Answer Justification :

Batholiths are the large body of magmatic material that cools in the deeper depth of the crust
develops in the form of large domes. They appear on the surface only after the denudational
processes remove the overlying materials. These are granitic bodies. Batholiths are the cooled
portion of magma chambers. Hence, statement 1 is correct.

prelims.insightsonindia.com 17
© Insights Active Learning | All rights reserved - 131815. You may not reproduce, distribute or exploit the contents in any form without
written permission by copyright owner. Copyright infringers may face civil and criminal liability
Total Marks : 200
Online Prelims TEST - 5 (SUBJECT WISE)
( InsightsIAS Mock Test Series for UPSC Preliminary Exam 2020 ) Mark Scored : 82

The near horizontal bodies of the intrusive igneous rocks are called sill or
sheet, depending on the thickness of the material. The thinner ones are called sheets while the
thick horizontal deposits are called sills. Hence, statement 2 is incorrect. Dykes are
perpendicular to the ground. Thus, vertical deposits.

Phacoliths are wavy mass of intrusive rocks, at times, is found at the base of synclines or at the
top of anticline in folded igneous country. Hence, statement 3 is correct.

30 Consider the following statements:


1. Mount Kilimanjaro is a volcanic cone located in Ethiopia.
2. Mount Kilimanjaro is the highest peak of African continent.
3. Kilimanjaro was recently in news for shrinking of glaciers.

Which of the statements given above is/are correct?


A. 3 only
4
03
B. 2 and 3 only
1
64
C. 1, 2 and 3
D. 1 and 2 only
5 4
- 87
om
Your Answer : B
Correct Answer : B
i l .c
m a
g
2@
Answer Justification :

m 2
pa cones, Kibo, Mawenzi, and Shira, is a dormant volcano
Mount Kilimanjaro with its three volcanic
u
in Tanzania. Hence, statementn1 is incorrect.
s a
da
- in Africa. Hence, statement 2 is correct.
It is the highest mountain
D as
The snows ofm
p a Kilimanjaro are rapidly disappearing and will be gone by 2033, predicts the

nu
most detailed analysis yet of the iconic glaciers gracing Africa's highest peak. Hence, statement
3 is A
correct.

https://www.livescience.com/41930-kilimanjaro-glaciers-shrinking.html

https://www.nature.com/articles/news.2009.1055

31 Consider the following statements:


1. The higher concentration of dust particles is found in Equatorial and Polar regions than subtropical
and temperate regions.
2. Water vapour generally decreases with altitude.

Which of the statements given above is/are correct?


A. 1 only
B. 2 only
C. Both 1 and 2
D. Neither 1 nor 2

prelims.insightsonindia.com 18
© Insights Active Learning | All rights reserved - 131815. You may not reproduce, distribute or exploit the contents in any form without
written permission by copyright owner. Copyright infringers may face civil and criminal liability
Total Marks : 200
Online Prelims TEST - 5 (SUBJECT WISE)
( InsightsIAS Mock Test Series for UPSC Preliminary Exam 2020 ) Mark Scored : 82

Your Answer : B
Correct Answer : B

Answer Justification :

The higher concentration of dust particles is found in subtropical and temperate regions
due to dry winds in comparison to equatorial and polar regions. Dust and salt particles act as
hygroscopic nuclei around which water vapour condenses to produce clouds. Hence,
statement 1 is incorrect.

Water vapour is a variable gas in the atmosphere, which decreases with altitude. Water vapour
also decreases from the equator towards the poles. It also absorbs parts of the insolation from
the sun and preserves the earth’s radiated heat. Hence, statement 2 is incorrect.

32 Consider the following statements:


1. Lake Michigan is located entirely within the USA. 4
2. Lake Baikal is the world’s largest freshwater lake. 1 03
3. River Blue Nile originates from Lake Victoria. 4 64
5
- 87
Which of the statements given above is/are correct?
A. 1 and 2 only . c om
B. 2 and 3 only a il
C. 1, 2 and 3
@ gm
D. 1 only 22 m
u pa
Your Answer :
a n
s
Correct Answer : A
da
a s-
D :
Answer Justification
m
u pa is located entirely within the USA. It is the second largest of the Great Lakes by
Lake Michigan
Anand the third largest the Great Lakes by area. Lake Michigan connects to Lake Huron
volume
through the Straits of Mackinac.

Hence, statement 1 is correct.

Lake Baikal is situated in southern Siberia and is the world’s largest freshwater lake,
accounting for about 22% of all fresh surface water in the world. It is the world’s second-largest
lake by volume. Lake Baikal is also the deepest lake in the world.

The Blue Nile is a river originating at Lake Tana in Ethiopia. With the White Nile, it is one of
the two major tributaries of the Nile. The Blue Nile joins the White Nile at Khartoum and, as
the Nile, flows through Egypt to the Mediterranean Sea at Alexandria. Hence, statement 3 is
incorrect.

33 Consider the following statements:


1. Mulching helps to retain the soil moisture.

prelims.insightsonindia.com 19
© Insights Active Learning | All rights reserved - 131815. You may not reproduce, distribute or exploit the contents in any form without
written permission by copyright owner. Copyright infringers may face civil and criminal liability
Total Marks : 200
Online Prelims TEST - 5 (SUBJECT WISE)
( InsightsIAS Mock Test Series for UPSC Preliminary Exam 2020 ) Mark Scored : 82

2. In the coastal and dry regions shelter belts are used to protect soil cover.

Which of the statements given above is/are correct?


A. 1 only
B. 2 only
C. Both 1 and 2
D. Neither 1 nor 2

Your Answer : C
Correct Answer : C

Answer Justification :

Mulching is a method of soil conservation where the bare ground between plants is covered
with a layer of organic matter like straw. It helps to retain soil moisture. Hence, statement
1 is correct.
0 34
1 regions in which
Shelter Belts is a method of soil conservation practiced in the coastal and4dry
6
rows of trees are planted to check the wind movement to protect soil
7 54cover. Hence,
statement 2 is correct.
-8
. c om
a
34 With reference to the ‘Continental Drift Theory’ proposed ilby Alfred Wegener, consider the following
statements:
@ gm
1. In the beginning, all the continents formed2a2single continental mass called Laurasia.
am of the continents was caused by pole-fleeing force and
2. The movement responsible for the drifting
p
tidal force. n u
a sa
d
Which of the statements- given above is/are correct?
s
A. 1 only Da
B. 2 onlyam
n up1 and 2
C. Both
D.ANeither 1 nor 2

Your Answer : C
Correct Answer : B

Answer Justification :

According to Wegener, all the continents formed a single continental mass and mega ocean
surrounded the same. The super continent was named PANGAEA, which meant all earth. The
mega-ocean was called PANTHALASSA, meaning all water. He argued that, around 200 million
years ago, the super continent, Pangaea, began to split. Pangaea first broke into two large
continental masses as Laurasia and Gondwanaland forming the northern and southern
components respectively. Hence, statement 1 is incorrect.

Wegener suggested that the movement responsible for the drifting of the continents was caused by
pole-fleeing force and tidal force. The polar-fleeing force relates to the rotation of the earth
and the tidal force is due to the attraction of the moon and the sun that develops tides in

prelims.insightsonindia.com 20
© Insights Active Learning | All rights reserved - 131815. You may not reproduce, distribute or exploit the contents in any form without
written permission by copyright owner. Copyright infringers may face civil and criminal liability
Total Marks : 200
Online Prelims TEST - 5 (SUBJECT WISE)
( InsightsIAS Mock Test Series for UPSC Preliminary Exam 2020 ) Mark Scored : 82

oceanic waters. Hence, statement 2 is correct.

35 Which of the following factors does/do cause variation in insolation?


1. The rotation of earth.
2. The angle of inclination of the sun’s rays
3. The length of the day

Select the correct answer using the code given below:


A. 1 and 2 only
B. 2 and 3 only
C. 1 and 3 only
D. 1, 2 and 3

Your Answer : B
Correct Answer : D
4
1 03
Answer Justification :
64
54
87 which in short is termed
The energy received by the earth is known as incoming solar radiation
-
as insolation. m
i l .co
that cause these variations in insolation are : g maa day, in a season and in a year. The factors
The amount and the intensity of insolation vary during

2 2@
(i) The rotation of earth on its axis am
n up
(ii) The angle of inclinationsofa the sun’s rays
a
s -d
Da
(iii) The length of the day

p am
(iv) The transparency of the atmosphere
u
An
(v) The configuration of land in terms of its aspect

Thus, all the given factors cause variation in the amount of insolation.

Hence, option (d) is correct.

36 Consider the following statements:


1. The convective transfer of energy is confined only to the troposphere.
2. The transfer of heat through horizontal movement of air is called advection.

Which of the statements given above is/are correct?


A. 1 only
B. 2 only
C. Both 1 and 2
D. Neither 1 nor 2.

prelims.insightsonindia.com 21
© Insights Active Learning | All rights reserved - 131815. You may not reproduce, distribute or exploit the contents in any form without
written permission by copyright owner. Copyright infringers may face civil and criminal liability
Total Marks : 200
Online Prelims TEST - 5 (SUBJECT WISE)
( InsightsIAS Mock Test Series for UPSC Preliminary Exam 2020 ) Mark Scored : 82

Your Answer :
Correct Answer : C

Answer Justification :

The air in contact with the earth rises vertically on heating in the form of currents and further
transmits the heat of the atmosphere. This process of vertical heating of the atmosphere is known
as convection. The convective transfer of energy is confined only to the troposphere. Hence,
statement 1 is correct.

The transfer of heat through horizontal movement of air is called advection. It plays important
role diurnal variation in daily weather in mid latitude and the local wind ‘loo’ in northern India.
Hence, statement 2 is correct.

37 With reference to the ‘South China Sea’, consider the following statements:
1. Malaysia and Singapore are bordering nations of South China Sea.
0 34
2. River Mekong drains into South China Sea. 1
4Sea.
4 6
3. Disputed islands the Paracel and Spratly islands are located in South China
7 5
Which of the statements given above is/are correct? -8
A. 1 and 2 only . c om
B. 2 and 3 only a il
C. 1 and 3 only
@ gm
D. 1, 2 and 3 22 m
u pa
Your Answer : D
a n
s
Correct Answer : D
da
a s-
D :
Answer Justification
a m
u
States and pterritories with borders on the sea include: China,
A n
the Philippines, Malaysia, Brunei, Indonesia, Singapore, and Vietnam. Hence, statement 1 is
correct.

Nine major rivers flow into the sea. Namely, these include the Min, Mekong, Pearl, Red, Pampanga,
Pahang, Pasig, and Jiulong Rivers. Hence, statement 2 is correct.

Disputed islands the Paracel and Spratly islands are located in South China Sea. Hence, statement
3 is correct.

38 Diclofenac, a painkiller used in livestock has emerged as major cause for decline in the population of:

A. Cattle
B. Tigers
C. Sparrows
D. Vultures

prelims.insightsonindia.com 22
© Insights Active Learning | All rights reserved - 131815. You may not reproduce, distribute or exploit the contents in any form without
written permission by copyright owner. Copyright infringers may face civil and criminal liability
Total Marks : 200
Online Prelims TEST - 5 (SUBJECT WISE)
( InsightsIAS Mock Test Series for UPSC Preliminary Exam 2020 ) Mark Scored : 82

Your Answer : D
Correct Answer : D

Answer Justification :

Vultures in the Indian subcontinent were dying of kidney failure shortly after scavenging
livestock treated with diclofenac, a painkiller that is similar to aspirin or ibuprofen. Government
has banned the usage of diclofenac for cattle. Hence, option(d) is correct.

39 Consider the following statements:


1. Convection currents are generated due to radioactive elements causing thermal differences in the
mantle portion.
2. The ocean crust rocks are much older than the continental rocks.
3. In the mid-oceanic ridge areas, the earthquake foci have shallow depths.

4
03
Which of the statements given above is/are correct?
A. 1 only 1
B. 2 and 3 only 4 64
5
C. 1, 2 and 3
- 87
om
D. 1 and 3 only
i l .c
Your Answer : B m a
g
2@
Correct Answer : D
2
am
up
Answer Justification :
an
as the possibility of convection currents operating in the mantle
Arthur Holmes in 1930s discussed
d
s - are generated due to radioactive elements causing thermal
portion. These currents
a
D mantle portion. Hence, statement 1 is correct.
differences in the

p am
u crust rocks are much younger than the continental rocks. The age of rocks in the
The ocean
n
A
oceanic crust is less than 200 million years old, while some of the continental rock formations are as
old as 3,200 million years. Hence, statement 2 is incorrect.

The deep trenches have deep-seated earthquake occurrences while in the mid-oceanic ridge
areas, the quake foci have shallow depths. Hence, statement 3 is correct.

40 With reference to the ‘Temperature Inversion’ mechanism in meteorology, consider the following
factors:
1. A long winter night
2. Cloudy sky
3. Still air

Which of the above given factors are ideal condition for temperature inversion?
A. 1 only
B. 2 and 3 only
C. 1, 2 and 3

prelims.insightsonindia.com 23
© Insights Active Learning | All rights reserved - 131815. You may not reproduce, distribute or exploit the contents in any form without
written permission by copyright owner. Copyright infringers may face civil and criminal liability
Total Marks : 200
Online Prelims TEST - 5 (SUBJECT WISE)
( InsightsIAS Mock Test Series for UPSC Preliminary Exam 2020 ) Mark Scored : 82

D. 1 and 3 only

Your Answer : C
Correct Answer : D

Answer Justification :

When the temperature increases with altitude, it is known as temperature inversion. A long winter
night with clear skies and still air is ideal situation for inversion. Hence, option (d) is correct
answer.

41 Consider the following statements:


1. The pressure gradient is weak where the isobars are close to each other.
2. Coriolis force deflects the wind to the left direction in the northern hemisphere.
3. The Coriolis force acts parallel to the pressure gradient force.
4
1 03
64
Which of the statements given above is/are correct?
A. 1 and 2 only
5 4
B. 1, 2 and 3
- 87
om
C. 3 only
D. None
i l .c
m a
g
2@
Your Answer : D
Correct Answer : D 2
am
up
Answer Justification :
s an
da
s - is caused due difference in the pressure between two regions.
Pressure Gradient Force
Da
am of pressure with respect to distance is the pressure gradient. The pressure
The rate of change
p
gradient
n uis strong where the isobars are close to each other and is weak where the isobars
A
are apart. Hence, statement 1 is incorrect.

The rotation of the earth about its axis affects the direction of the wind. This force is called the
Coriolis force. It deflects the wind to the right direction in the northern hemisphere and to
the left in the southern hemisphere. Hence, statement 2 is incorrect.

The Coriolis force acts perpendicular to the pressure gradient force. Hence, statement 3 is
incorrect.

42 Consider the following statements:Tibetan Plateau is an intermontane plateau.


1. Tibetan Plateau is an intermontane plateau.
2. Columbia Plateau is a volcanic plateau.

Which of the statements given above is/are correct?


A. 1 only
B. 2 only

prelims.insightsonindia.com 24
© Insights Active Learning | All rights reserved - 131815. You may not reproduce, distribute or exploit the contents in any form without
written permission by copyright owner. Copyright infringers may face civil and criminal liability
Total Marks : 200
Online Prelims TEST - 5 (SUBJECT WISE)
( InsightsIAS Mock Test Series for UPSC Preliminary Exam 2020 ) Mark Scored : 82

C. Both 1 and 2
D. Neither 1 nor 2.

Your Answer : C
Correct Answer : C

Answer Justification :

The highest plateaus in the world, bordering the mountains are referred to as intermontane
plateaus. Example includes the Tibetan Plateau in Asia which is also the largest plateau in the
world. Hence, statement 1 is correct.

A volcanic plateau is a type of plateau that is formed through volcanic activities such as the
extrusion of lava or upwelling of magma. Some of the examples of volcanic plateaus include North
Island Volcanic Plateau, Columbia Plateau, and Shirasu-Daichi. Hence, statement 2 is correct.

4
43 Which of the following is/are minor tectonic plates? 1 03
1. Antarctica Plate 4 64
5
2. Nazca Plate
- 87
3. Cocos Plate
4. Philippine Plate .com
a i l
g
Select the correct answer using the code given below:
m
2 @
m2
A. 1, 3 and 4 only
a
up
B. 2, 3 and 4 only
C. 2 and 3 only
a n
D. 1 and 4 only as d
-
Your Answer : D as
am: B
Correct Answer
nup
A
Answer Justification :

The major tectonic plates are Antarctica and the surrounding oceanic plate, North American
Plate, South American Plate, Pacific Plate, African Plate, Indo-Australian Plate, and Eurasian Plate.
Hence, Antarctic Plate is not a minor plate.

Some important minor plates are:


(i) Cocos plate : Between Central America and Pacific plate
(ii) Nazca plate : Between South America and Pacific plate
(iii) Arabian plate : Mostly the Saudi Arabian landmass
(iv) Philippine plate: Between the Asiatic and Pacific plate.

Hence, option (b) is correct answer.

44 Consider the following statements:


1. For the formation of dew, dew point should be below the freezing point.

prelims.insightsonindia.com 25
© Insights Active Learning | All rights reserved - 131815. You may not reproduce, distribute or exploit the contents in any form without
written permission by copyright owner. Copyright infringers may face civil and criminal liability
Total Marks : 200
Online Prelims TEST - 5 (SUBJECT WISE)
( InsightsIAS Mock Test Series for UPSC Preliminary Exam 2020 ) Mark Scored : 82

2. For the formation of the frost, dew point should be at or above the freezing point.

Which of the statements given above is/are correct?


A. 1 only
B. 2 only
C. Both 1 and 2
D. Neither 1 nor 2.

Your Answer : D
Correct Answer : D

Answer Justification :

When the moisture is deposited in the form of water droplets on cooler surfaces of solid objects, it is
known as dew. The ideal conditions for its formation are clear sky, calm air, high relative humidity,
and cold and long nights.
For the formation of dew, it is necessary that the dew point is above the freezing
0 34 point.
Hence, statement 1 is incorrect. 41 6
4
75
Frost forms on cold surfaces when condensation takes place below
- 8 freezing point (00C), i.e.
the dew point is at or below the freezing point. The excess
o mmoisture is deposited in the form of
l. c
minute ice crystals instead of water droplets. Hence, statement 2 is incorrect.
i
m a
@g
22 for warm/hot wind?
45 Which of the following local winds is an example
m
u pa
A. Chinook
a n
s
B. Pampero
- da
C. Mistral s
D. Bora Da
p am
nu : A
Your Answer
A
Correct Answer : A

Answer Justification :

Chinook, warm, dry wind descending the eastern slopes of the Rocky Mountains, primarily in
winter. Winds of the same kind occur in other parts of the world and are known generally as foehns.

Other three local winds given in the option are cold winds.

Hence, option (a) is correct.

46 Kiruna and Gallivare located in Sweden are well known for

A. Oil deposits
B. Iron mines
C. Automobile industry.

prelims.insightsonindia.com 26
© Insights Active Learning | All rights reserved - 131815. You may not reproduce, distribute or exploit the contents in any form without
written permission by copyright owner. Copyright infringers may face civil and criminal liability
Total Marks : 200
Online Prelims TEST - 5 (SUBJECT WISE)
( InsightsIAS Mock Test Series for UPSC Preliminary Exam 2020 ) Mark Scored : 82

D. Coal depositsCoal deposits

Your Answer :
Correct Answer : B

Answer Justification :

The Kiruna mine is the largest and most modern underground iron ore mine in the world. The
mine is located in Kiruna in Norrbotten County, Lapland, Sweden.

Gällivare is located in a major iron ore mining region.

Hence, option (b) is correct.

47 Consider the following statements:

34
1. River Amazon drains into the Pacific Ocean.
1 0
2. St. Louis is located at the confluence of the River Mississippi and the River Missouri.
3. The Grand Igna Dam is a proposed hydroelectric dam on the Congo River.64
5 4
Which of the statements given above is/are correct? - 87
A. 1 and 2 only
. c om
B. 2 and 3 only a il
C. 1 and 3 only
@ gm
D. 1, 2 and 3 22 m
u pa
Your Answer :
a n
s
da
Correct Answer : B

s-
Da :
Answer Justification
am
up River in South America is the largest river by discharge volume of water in the world.
The Amazon
n
The A
Amazon enters Brazil with only one-fifth of the flow it finally discharges into the Atlantic
Ocean. Hence, statement 1 is incorrect.

The Mississippi River is the second-longest river and chief river of the second-largest drainage
system on the North American continent. It joins the River Missouri at St. Louis. Hence,
statement 2 is correct.

The Grand Inga Dam is a proposed hydroelectric dam on the Congo River at Inga Falls in the
Democratic Republic of the Congo. Hence, statement 3 is correct.

48 Consider the following statements:


1. The Mid-Atlantic Ridge is an example of divergent boundary.
2. At transform boundaries crust is neither created nor destroyed.

Which of the statements given above is/are correct?


A. 1 only

prelims.insightsonindia.com 27
© Insights Active Learning | All rights reserved - 131815. You may not reproduce, distribute or exploit the contents in any form without
written permission by copyright owner. Copyright infringers may face civil and criminal liability
Total Marks : 200
Online Prelims TEST - 5 (SUBJECT WISE)
( InsightsIAS Mock Test Series for UPSC Preliminary Exam 2020 ) Mark Scored : 82

B. 2 only
C. Both 1 and 2
D. Neither 1 nor 2.

Your Answer : C
Correct Answer : C

Answer Justification :

At divergent boundary, new crust is generated as the plates pull away from each other. The best-
known example of divergent boundaries is the Mid-Atlantic Ridge. At this, the American
Plates (South America and North America) are separated from the Eurasian and African
Plates. Hence, statement 1 is correct.

At transform boundaries, the crust is neither produced nor destroyed as the plates slide
horizontally past each other. Transform faults are the planes of separation generally
perpendicular to the mid-oceanic ridges. Hence, statement 2 is correct. 34 0
6 41
4
75recent volcanic eruption is
49 In which of the following countries, the mount Sinabung which has seen
8
located? -
o m
A. Malaysia a il.c
m
B. Indonesia
2 @g
C. Japan
m 2
D. Myanmar
u pa
a n
s
Your Answer : A
- da
Correct Answer : B s
Da
am
Answer Justification :
nup
A Sinabung is a Pleistocene-to-Holocene strato volcano located in the North Sumatra,
Mount
Indonesia.

https://www.express.co.uk/news/world/1138564/Indonesia-volcano-eruption-mount-sinabung-latest-a
sh-cloud-sumatra

50 ‘Ladang’ is a local name of shifting cultivation practiced in:

A. Southeast Asia
B. Africa
C. Central America
D. Europe

Your Answer :
Correct Answer : A

prelims.insightsonindia.com 28
© Insights Active Learning | All rights reserved - 131815. You may not reproduce, distribute or exploit the contents in any form without
written permission by copyright owner. Copyright infringers may face civil and criminal liability
Total Marks : 200
Online Prelims TEST - 5 (SUBJECT WISE)
( InsightsIAS Mock Test Series for UPSC Preliminary Exam 2020 ) Mark Scored : 82

Answer Justification :

Shifting cultivation is widely practiced by many tribes in the tropics, especially in Africa, South and
Central America and southeast Asia. It is prevalent in tropical region in different names, e.g.
Jhuming in North eastern states of India, Milpa in Central America and Mexico and
Ladang in Indonesia and Malaysia. Hence, option (a) is correct answer.

51 The River Orange forms the international boundary between which of the following countries?

A. Niger and Nigeria


B. South Africa and Namibia
C. South Africa and Zimbabwe
D. Nigeria and Cameroon

Your Answer :
4
03
Correct Answer : B
1
Answer Justification : 4 64
5
87
- the continent and one of the
Orange River, river in southern Africa, one of the longest rivers on
m
longest south of the Tropic of Capricorn. .co il
m a
g
Along its course the river forms the boundary between Namibia and South Africa.
@
2 2
Hence, option (b) is correct. m
u pa
n
sa
da
52 Consider the following pairs:
-
Name of the Dam
D as River
1. Kariba Dam Caroni
a m
n up
Grand Coulee Columbia
A
Guri Dam Zambezi

Which of the pairs given above is/are correctly matched?


A. 1 and 2 only
B. 1 and 3 only
C. 2 only
D. 3 only

Your Answer :
Correct Answer : C

Answer Justification :

The Kariba Dam is a double curvature concrete arch dam in the Kariba Gorge of the Zambezi
river basin between Zambia and Zimbabwe. Hence, pair 1 is not correctly matched.

prelims.insightsonindia.com 29
© Insights Active Learning | All rights reserved - 131815. You may not reproduce, distribute or exploit the contents in any form without
written permission by copyright owner. Copyright infringers may face civil and criminal liability
Total Marks : 200
Online Prelims TEST - 5 (SUBJECT WISE)
( InsightsIAS Mock Test Series for UPSC Preliminary Exam 2020 ) Mark Scored : 82

Grand Coulee Dam is a concrete gravity dam on the Columbia River in the U.S. state of
Washington, built to produce hydroelectric power and provide irrigation water. Hence, pair 2 is
correctly matched.

The Simón Bolívar Hydroelectric Plant, also Guri Dam, is a concrete gravity and embankment dam
in Bolívar State, Venezuela on the Caroni River. Hence, pair 3 is not correctly matched.

53 Which of the following elements makes the highest share in the Earth’s crust by weight?

A. Silicon
B. Iron
C. Aluminum
D. Oxygen

Your Answer : D
4
03
Correct Answer : D
1
Answer Justification : 4 64
5
- 87
The Major Elements of the Earth’s Crust:
.com
Elements By Weight(%) a i l
g m
1. Oxygen 46.60
2 2@
am
2. Silicon 27.72 up
s an
3. Aluminum 8.13
- da
s
4. Iron 5.00 Da
p am
u 3.63
An
5. Calcium

6. Sodium 2.83

7. Potassium 2.59

8. Magnesium 2.09

9. Others 1.41

Hence, option (d) is correct.

54 With reference to the ‘Savanna Climate’, consider the following statements:


1. Savanna climate has alternate wet and dry seasons.
2. It is known for commercial grain farming.
3. Masai tribes who inhabit savanna region are pastoralists.

prelims.insightsonindia.com 30
© Insights Active Learning | All rights reserved - 131815. You may not reproduce, distribute or exploit the contents in any form without
written permission by copyright owner. Copyright infringers may face civil and criminal liability
Total Marks : 200
Online Prelims TEST - 5 (SUBJECT WISE)
( InsightsIAS Mock Test Series for UPSC Preliminary Exam 2020 ) Mark Scored : 82

Which of the statements given above is/are correct?


A. 1 and 3 only
B. 1 and 2 only
C. 2 and 3 only
D. 1, 2 and 3

Your Answer :
Correct Answer : A

Answer Justification :

Savanna climate has alternate wet and dry seasons. There are only two seasons – winter and
summer. Rains occur in summer. Hence, statement 1 is correct

Due to distinct wet and dry periods, many Savanna areas have poor lateritic soils which are
incapable of supporting good crops. Temperate grasslands are known for commercial grain
farming. Hence, statement 2 is incorrect.
0 34
6 41
5 4
The Masai tribes of savanna are pastoralists are of the East African plateau. They are known for
7
-8
cattle rearing. Hence, statement 3 is correct.

. c om
55 In which of the following regions do Bushmen tribe are a il
found?

@ gm
A. Atacama Desert 2 2
B. Kalahari Desert
pam
C. Mojave Desert n u
D. Thar Desert a sa
s -d
Your Answer : Da
am: B
Correct Answer
p
n u
A
Answer Justification :

The Bushmen are the indigenous peoples of southern Africa who are largely hunter-
gatherers. They live in the region of Kalahari Desert.

Hence, option (b) is correct.

56 Consider the following statements:


1. Canyons and trenches are observed in the continental slope.
2. Seamounts are volcanic in origin.

Which of the statements given above is/are correct?


A. 1 only
B. 2 only
C. Both 1 and 2
D. Neither 1 nor 2.

prelims.insightsonindia.com 31
© Insights Active Learning | All rights reserved - 131815. You may not reproduce, distribute or exploit the contents in any form without
written permission by copyright owner. Copyright infringers may face civil and criminal liability
Total Marks : 200
Online Prelims TEST - 5 (SUBJECT WISE)
( InsightsIAS Mock Test Series for UPSC Preliminary Exam 2020 ) Mark Scored : 82

Your Answer :
Correct Answer : C

Answer Justification :

The continental slope connects the continental shelf and the ocean basins. It begins where
the bottom of the continental shelf sharply drops off into a steep slope. The slope boundary
indicates the end of the continents. Canyons and trenches are observed in this region. Hence,
statement 1 is correct.

Seamount is a mountain with pointed summits, rising from the seafloor that does not reach the
surface of the ocean. Seamounts are volcanic in origin. These can be 3,000-4,500 m tall. Hence,
statement 2 is correct.

57 Consider the following statements:


1. Russia shares border with Baltic Sea and Caspian Sea. 4
2. Sea of Galilee is a freshwater lake located in Israel. 1 03
4 64
5
Which of the statements given above is/are correct?
- 87
A. 1 only
B. 2 only . c om
C. Both 1 and 2 a il
D. Neither 1 nor 2. gm @
2 2
m
Your Answer :
u pa
Correct Answer : C
a n
s
Answer Justification :-
da
s
Da
am Azerbaijan, Kazakhstan, Russia, Turkmenistan and Iran share the natural
The five countries
p
heritageuof the Caspian Sea.
An
The Baltic Sea is positioned in Northern Europe and bordered by Sweden, Finland, Russia,
Estonia, Latvia, Lithuania, Poland, northeastern Germany, and eastern Denmark and its
numerous islands. Hence, statement 1 is correct.

The Sea of Galilee is a freshwater lake located in Northern Israel. The Jordan River is the
main inlet into the Sea of Galilee. It is surrounded by Golan Heights in the east. The Sea of Galilee
is the second lowest-lying lake in the world after the Dead Sea. Hence, statement 2 is correct.

58 Consider the following statements:


1. Quartz is used in radio and radar.
2. Pyroxene is commonly found in meteorites.

Which of the statements given above is/are correct?


A. 1 only
B. 2 only

prelims.insightsonindia.com 32
© Insights Active Learning | All rights reserved - 131815. You may not reproduce, distribute or exploit the contents in any form without
written permission by copyright owner. Copyright infringers may face civil and criminal liability
Total Marks : 200
Online Prelims TEST - 5 (SUBJECT WISE)
( InsightsIAS Mock Test Series for UPSC Preliminary Exam 2020 ) Mark Scored : 82

C. Both 1 and 2
D. Neither 1 nor 2.

Your Answer :
Correct Answer : C

Answer Justification :

Quartz is one of the most important components of sand and granite. It consists of silica. It is a
hard mineral virtually insoluble in water. It is white or colourless and used in radio and
radar. Hence, statement 1 is correct.

Pyroxene consists of calcium, aluminum, magnesium, iron and silica. Pyroxene forms 10 per cent of
the earth’s crust. It is commonly found in meteorites. Hence, statement 2 is correct.

59 Consider the following statements:


0 34
1
1. The enclosed seas in the low latitudes have lower temperature than the open seas.
64 seas
2. The enclosed seas in the high latitudes have higher temperature than the4open
7 5
Which of the statements given above is/are correct? -8
A. 1 only . c om
B. 2 only a il
C. Both 1 and 2
@ gm
D. Neither 1 nor 2. 22 m
u pa
Your Answer :
a n
s
Correct Answer : D
da
a s-
D :
Answer Justification
m
u pa seas in the low latitudes record relatively higher temperature than the open
The enclosed
n
seasAwhereas the enclosed seas in the high latitudes have lower temperature than the open
seas.

Hence, both statement 1 and statement 2 are incorrect.

60 Lake Van, one of the highest saline lake in the world is located in:

A. U.S.A
B. Egypt
C. Sudan
D. Turkey

Your Answer :
Correct Answer : D

Answer Justification :

prelims.insightsonindia.com 33
© Insights Active Learning | All rights reserved - 131815. You may not reproduce, distribute or exploit the contents in any form without
written permission by copyright owner. Copyright infringers may face civil and criminal liability
Total Marks : 200
Online Prelims TEST - 5 (SUBJECT WISE)
( InsightsIAS Mock Test Series for UPSC Preliminary Exam 2020 ) Mark Scored : 82

Lake Van, Turkish Van Gölü, lake, largest body of water in Turkey and the second largest in
the Middle East. The lake is located in the region of eastern Anatolia near the border of Iran.

Hence, option (d) is correct.

61 Consider the following statements:


1. Spring tides occur when the sun, the moon and the earth are in a straight line.
2. Neap tides occur when the sun and moon are at right angles to each other.

Which of the statements given above is/are correct?


A. 1 only
B. 2 only
C. Both 1 and 2
D. Neither 1 nor 2.

4
03
Your Answer : C
1
64
Correct Answer : C
5 4
Answer Justification :
- 87
m
When the sun, the moon and the earth are in a straight
i l .coline, the height of the tide will be
higher. These are called spring tides and they occuratwice a month, one on full moon period and
another during new moon period. Hence, statement
@ gm 1 is correct.
2 2
Neap tides occur when the sun and moon
p am are at right angles to each other and the forces of
u
an
the sun and moon tend to counteract one another. . Hence, statement 2 is correct.
s
- da
s
62 Which of the followingaindustries is/are footloose industries?
1. Cotton textilesm
D
pa
2. Cement industry
u
Anmanufacturing
3. Mobile
4. Iron and Steel industry

Select the correct answer using the code given below:


A. 1, 2 and 4 only
B. 1, 3 and 4 only
C. 1 and 3 only
D. 3 only

Your Answer :
Correct Answer : D

Answer Justification :

Foot loose industries can be located in a wide variety of places. They are not dependent on any
specific raw material, weight losing or otherwise. They largely depend on component parts
which can be obtained anywhere. They produce in small quantity and also employ a small labour

prelims.insightsonindia.com 34
© Insights Active Learning | All rights reserved - 131815. You may not reproduce, distribute or exploit the contents in any form without
written permission by copyright owner. Copyright infringers may face civil and criminal liability
Total Marks : 200
Online Prelims TEST - 5 (SUBJECT WISE)
( InsightsIAS Mock Test Series for UPSC Preliminary Exam 2020 ) Mark Scored : 82

force. These are generally not polluting industries. The important factor in their location is
accessibility by road network. Diamonds, computer chips, and mobile manufacturing are
some examples of footloose industries.

Location of Cotton textiles, Cement industry and, Iron and Steel industries largely depend
upon location of the raw materials. Thus, cannot be considered under the category of the
Footloose industries.

Hence, option (d) is correct answer.

63 Tuff, gabbro and pegmatite are examples for:

A. Igneous Rocks
B. Sedimentary Rocks
C. Metamorphic Rocks
4
03
D. None
1
Your Answer : C 4 64
5
Correct Answer : A
- 87
Answer Justification : .com
a i l
gmthe interior of the earth, thus they are known
The Igneous rocks form out of magma and lava from
@
2 2 basalt, volcanic breccia and tuff are some of the
as primary rocks. Granite, gabbro, pegmatite,
examples of igneous rocks. am p
u
an
as
Hence, option (a) is correct.
d
-
as
D Ocean currents is a warm current?
64 Which of the following
p am
nu
A.AKuroshio Current
B. Oyashio Current
C. California Current
D. Benguela Current.

Your Answer : A
Correct Answer : A

Answer Justification :

prelims.insightsonindia.com 35
© Insights Active Learning | All rights reserved - 131815. You may not reproduce, distribute or exploit the contents in any form without
written permission by copyright owner. Copyright infringers may face civil and criminal liability
Total Marks : 200
Online Prelims TEST - 5 (SUBJECT WISE)
( InsightsIAS Mock Test Series for UPSC Preliminary Exam 2020 ) Mark Scored : 82

4
1 03
4 64
Hence, option (a) is correct. 5
- 87
65 Sunda strait connects
.com
a i l
m
A. Indian Ocean and Java Sea
2 @g
m2
B. Java Sea and South China Sea
a
up
C. Java Sea and Celebes Sea
D. Philippines Sea and PacificnSea
s a
- da
Your Answer : s
Aa
Correct Answer : D

p am
uJustification :
An
Answer

The Sunda Strait is the strait between the Indonesian islands of Java and Sumatra. It connects the
Java Sea to the Indian Ocean. Hence, option (a) is correct.

prelims.insightsonindia.com 36
© Insights Active Learning | All rights reserved - 131815. You may not reproduce, distribute or exploit the contents in any form without
written permission by copyright owner. Copyright infringers may face civil and criminal liability
Total Marks : 200
Online Prelims TEST - 5 (SUBJECT WISE)
( InsightsIAS Mock Test Series for UPSC Preliminary Exam 2020 ) Mark Scored : 82

4
1 03
4 64
5
- 87
.com
66 Pine, Spruce and Fir trees are found in:
a i l
gm
A. Savanna Forests
2 2@
B. Taiga Forests m
C. Tropical Hardwood deciduousu pa
n
D. sa
Equatorial Evergreen Forests
a
d
Your Answer : D a s-
D
m
Correct Answer : B
u pa
An Justification :
Answer

Taiga, generally referred to in North America as boreal forest or snow forest, is a biome
characterized by coniferous forests consisting mostly of fir, pines, spruces, and larches. The
taiga or boreal forest is the world's largest land biome. Hence, option (b) is correct.

67 Which of the following countries is a landlocked country?

A. Nepal
B. Myanmar
C. Bangladesh
D. Thailand

Your Answer : A
Correct Answer : A

prelims.insightsonindia.com 37
© Insights Active Learning | All rights reserved - 131815. You may not reproduce, distribute or exploit the contents in any form without
written permission by copyright owner. Copyright infringers may face civil and criminal liability
Total Marks : 200
Online Prelims TEST - 5 (SUBJECT WISE)
( InsightsIAS Mock Test Series for UPSC Preliminary Exam 2020 ) Mark Scored : 82

Answer Justification :

4
1 03
4 64
5
- 87
.com
a i l
g m
Hence, option (a) is correct.
2 2@
am
up
s an
da
s-
68 Consider the following statements:
a
1. Weathering is an D
in-situ or on-site process.
m
pa turns into gypsum by the process of hydration.
2. Calcium sulphate
A nu
Which of the statements given above is/are correct?
A. 1 only
B. 2 only
C. Both 1 and 2
D. Neither 1 nor 2.

Your Answer : B
Correct Answer : C

Answer Justification :

Weathering is defined as mechanical disintegration and chemical decomposition of rocks


through the actions of various elements of weather and climate. As very little or no motion of
materials takes place in weathering, it is an in-situ or on-site process. Hence, statement 1 is
correct.

prelims.insightsonindia.com 38
© Insights Active Learning | All rights reserved - 131815. You may not reproduce, distribute or exploit the contents in any form without
written permission by copyright owner. Copyright infringers may face civil and criminal liability
Total Marks : 200
Online Prelims TEST - 5 (SUBJECT WISE)
( InsightsIAS Mock Test Series for UPSC Preliminary Exam 2020 ) Mark Scored : 82

Hydration is the chemical addition of water. Minerals take up water and expand and expansion
causes an increase in the volume of the material itself or rock. Calcium sulphate takes in water
and turns to gypsum, which is
more unstable than calcium sulphate. Hence, statement 2 is correct.

69 Among which of the following countries the tropic of Capricorn does not pass through?

A. Argentina
B. Australia
C. Namibia
D. Zimbabwe

Your Answer : B
Correct Answer : D

4
Answer Justification :
1 03
4 64
5
- 87
.com
a i l
g m
2 2@
am
up
s an
- da
s
Da
m
u pa
An

Hence, option (d) is correct.

70 The famous fishing ground ‘Dogger Bank’, is located in:

A. North Sea
B. Gulf of Mexico
C. Black Sea
D. Gulf of Mannar

Your Answer : B

prelims.insightsonindia.com 39
© Insights Active Learning | All rights reserved - 131815. You may not reproduce, distribute or exploit the contents in any form without
written permission by copyright owner. Copyright infringers may face civil and criminal liability
Total Marks : 200
Online Prelims TEST - 5 (SUBJECT WISE)
( InsightsIAS Mock Test Series for UPSC Preliminary Exam 2020 ) Mark Scored : 82

Correct Answer : A

Answer Justification :

Dogger Bank, George Bank and Grand Bank are famous fishing grounds Of Atlantic Ocean. Dogger
Bank is a large sandbank in a shallow area of the North Sea. Hence, option(a) is correct.

71 Consider the following pairs:


Industrial Town Country
1. Detroit U.S.A.
2. Manchester U.K.
3. Baku Qatar

Which of the pairs given above is/are correctly matched?


4
A. 1 and 2 only
1 03
B. 2 and 3 only
4 64
5
87
C. 1 and 3 only
D. 2 only -
.com
Your Answer : A
a i l
Correct Answer : A g m
2 2@
Answer Justification :
am
n up
sa state of U.S.A. It is known for automobile industry. Hence, pair
Detroit is located in the Michigan
a
1 is correctly matched. d
a s-
D city in the northwest of England with a rich industrial heritage. It is famous
Manchester is a major
for textile p a m
industry. Hence, pair 2 is correctly matched
u
Baku,Anthe capital and commercial hub of Azerbaijan. Hence, pair 3 is not correctly matched.

72 With reference to the ‘Weathering process’, consider the following statements:


1. It helps in fastening the erosional process.
2. It has no role in the formation of soils.
3. It helps in the enrichment and concentrations of certain valuable ores.

Which of the statements given above is/are correct?


A. 1 only
B. 1 and 3 only
C. 2 only
D. 1, 2 and 3

Your Answer : B
Correct Answer : B

prelims.insightsonindia.com 40
© Insights Active Learning | All rights reserved - 131815. You may not reproduce, distribute or exploit the contents in any form without
written permission by copyright owner. Copyright infringers may face civil and criminal liability
Total Marks : 200
Online Prelims TEST - 5 (SUBJECT WISE)
( InsightsIAS Mock Test Series for UPSC Preliminary Exam 2020 ) Mark Scored : 82

Answer Justification :

Weathering processes are responsible for breaking down the rocks into smaller fragments
and preparing the way for formation of not only regolith and soils, but also erosion and
mass movements. Hence, statement 1 is correct but statement 2 is incorrect.

When rocks undergo weathering, some materials are removed through chemical or physical
leaching by groundwater and thereby the concentration of remaining (valuable) materials
increases. Thus, weathering of rocks and deposits helps in the enrichment and concentrations
of certain valuable ores of iron, manganese, aluminum, copper etc. Hence, statement 3 is
correct.

73 With reference to ‘Sargasso Sea’, consider then following statements:


#94000
1. It has no land boundary.
2. It is known for Sargassum seaweed. 4
1 03
Which of the statements given above is/are correct? 4 64
5
A. 1 only
B. 2 only - 87
C. Both 1 and 2 .com
D. Neither 1 nor 2 a i l
g m
Your Answer : C 2 2@
Correct Answer : C am
up
s an
da
Answer Justification :

The Sargasso Sea a


s-
D is a region of the North Atlantic Ocean bounded by four currents forming an
m all other regions called seas, it has no land boundaries. It is bounded on
ocean gyre. Unlike
a
p
the westuby the Gulf Stream; on the north, by the North Atlantic Current; on the east, by the Canary
An and on the south, by the North Atlantic Equatorial Current. Hence, statement 1 is
Current;
correct.

It is distinguished from other parts of the Atlantic Ocean by its characteristic brown Sargassum
seaweed. Hence, statement 2 is correct.

74 Which of the following countries are claiming territorial rights over Senkaku Island?

A. Japan and China


B. China and Vietman
C. Myanmar and Thailand
D. Japan and Russia

Your Answer : B
Correct Answer : A

prelims.insightsonindia.com 41
© Insights Active Learning | All rights reserved - 131815. You may not reproduce, distribute or exploit the contents in any form without
written permission by copyright owner. Copyright infringers may face civil and criminal liability
Total Marks : 200
Online Prelims TEST - 5 (SUBJECT WISE)
( InsightsIAS Mock Test Series for UPSC Preliminary Exam 2020 ) Mark Scored : 82

Answer Justification :

The Senkaku Islands dispute, or Diaoyu Islands dispute, concerns a territorial dispute over a
group of uninhabitedislands known as the Senkaku Islands in Japan, the Diaoyu Islands in China.

Hence, option (a) is correct.

75 Which of the following ports is/are examples of Ports of call?


1. Aden
2. Honolulu
3. Singapore

Select the correct answer using the code given below:


A. 1 only
B. 1 and 2 only
C. 1, 2 and 3 4
D. 3 only 1 03
4 64
5
Your Answer :
- 87
Correct Answer : C
.com
Answer Justification : a i l
@ gm
2
Ports of call are the ports which originally 2
developed as calling points on main sea routes where
am and taking food items. Later on, they developed into
ships used to anchor for refuelling, watering
p
u
an
commercial ports. Aden, Honolulu and Singapore are good examples.
s
- da
Hence, option (c) is correct.
s
Da
am statements:
76 Consider thepfollowing
u
An canal connects the Atlantic Ocean to the Pacific Ocean.
1. Panama
2. Suez Canal links the Mediterranean Sea and the Red Sea.

Which of the statements given above is/are correct?


A. 1 only
B. 2 only
C. Both 1 and 2
D. Neither 1 nor 2

Your Answer : C
Correct Answer : C

Answer Justification :

Panama canal connects the Atlantic Ocean in the east to the Pacific Ocean in the west. It has
been constructed across the Panama Isthmus between Panama City and Colon. Hence, statement 1
is correct.

prelims.insightsonindia.com 42
© Insights Active Learning | All rights reserved - 131815. You may not reproduce, distribute or exploit the contents in any form without
written permission by copyright owner. Copyright infringers may face civil and criminal liability
Total Marks : 200
Online Prelims TEST - 5 (SUBJECT WISE)
( InsightsIAS Mock Test Series for UPSC Preliminary Exam 2020 ) Mark Scored : 82

Suez canal had been constructed in 1869 in Egypt between Port Said in the north and Port Suez in
the south linking the Mediterranean Sea and the Red Sea. Hence, statement 2 is correct.

77 Which of the following activities belong to the Quaternary activities in the service sector of the
economy?
1. Information-generation and sharing
2. Medical Tourism
3. Research and Development

Select the correct answer using the code given below:


A. 1 only
B. 1 and 3 only
C. 2 and 3 only
D. 3 only

4
Your Answer : C
1 03
64
Correct Answer : B
5 4
Answer Justification :
- 87
.c
Quaternary activities involve some of the following: the collection, om
production and
i l
a of information. Quaternary activities centre
dissemination of information or even the production
g m
around research, development and may be seen @ as an advanced form of services involving
specialized knowledge and technical skills.22
pam
u
an tertiary sector of the economy, not Quaternary activities.
Medical tourism is classified into
s
- da
Hence, option (b) is correct.
s
Da
78 The famous p amCoal-field is located in:
Ruhr
u
An
A. Canada
B. Russia
C. Sweden
D. Germany

Your Answer : D
Correct Answer : D

Answer Justification :

The Ruhr is a large coal field located in the west of Germany. Ruhr represents one of the largest
coal reserves in Germany.

Hence, option (d) is correct.

79 Consider the following statements:

prelims.insightsonindia.com 43
© Insights Active Learning | All rights reserved - 131815. You may not reproduce, distribute or exploit the contents in any form without
written permission by copyright owner. Copyright infringers may face civil and criminal liability
Total Marks : 200
Online Prelims TEST - 5 (SUBJECT WISE)
( InsightsIAS Mock Test Series for UPSC Preliminary Exam 2020 ) Mark Scored : 82

1. Genetic biodiversity refers to the variation of genes within species.


2. Different human races are example of genetic diversity.

Which of the statements given above is/are correct?


A. 1 only
B. 2 only
C. Both 1 and 2
D. Neither 1 nor 2

Your Answer : C
Correct Answer : C

Answer Justification :

Genetic biodiversity refers to the variation of genes within species. Hence, statement 1 is correct.

34characteristics
Human beings genetically belong to the homo sapiens group and also differ in their
0
41 to genetic
such as height, colour, physical appearance, etc., considerably. This is due
6
diversity. Hence, statement 2 is correct. 54 7
-8
m
80 Consider the following statements:
i l .co
ma
1. The Convention of Biodiversity was signed at the Earth Summit held at Rio de Janeiro, Brazil in
1992.
@ g
22
2. IUCN publishes Red list of threatened species.
pam
u
n is/are correct?
Which of the statements given above
s a
A. 1 only
B. 2 only - da
s
C. Both 1 and D2 a
D. Neither
p am1 nor 2
nu
YourAAnswer : C
Correct Answer : C

Answer Justification :

The Government of India along with 155 other nations have signed the Convention of Biodiversity at
the Earth Summit held at Rio de Janeiro, Brazil in June 1992. Hence, statement 1 is correct.

The IUCN Red List of Threatened Species, founded in 1964, is the world's most comprehensive
inventory of the global conservation status of biological species. Hence, statement 2 is correct.

81 With reference to the Measles, consider the following statements:


1. It is a highly contagious bacterial disease.
2. It is transmitted via droplets from the nose, mouth or throat of infected persons.
3. Recently, India was declared measles free by World Health Organization (WHO).

prelims.insightsonindia.com 44
© Insights Active Learning | All rights reserved - 131815. You may not reproduce, distribute or exploit the contents in any form without
written permission by copyright owner. Copyright infringers may face civil and criminal liability
Total Marks : 200
Online Prelims TEST - 5 (SUBJECT WISE)
( InsightsIAS Mock Test Series for UPSC Preliminary Exam 2020 ) Mark Scored : 82

Which of the statement given above is/are correct?


A. 1 and 3 only
B. 2 only
C. 1, 2 and 3
D. 2 and 3 only

Your Answer : B
Correct Answer : B

Answer Justification :

Measles is a highly contagious viral disease. It remains an important cause of death among
young children globally, despite the availability of a safe and effective vaccine. Hence, statement
1 is incorrect.

Measles is transmitted via droplets from the nose, mouth or throat of infected persons. Hence,
statement 2 is correct. 34 0
6 41
https://www.who.int/immunization/diseases/measles/en/
5 4
7
-8
Under the Global Vaccine Action Plan, measles and rubella are targeted for elimination in five WHO
m
Regions by 2020. .co il
m a
Recently, Sri Lanka was declared measles free bygWorld Health Organization (WHO). With Sri
2 2@
Lanka’s recent achievement, five countries of the WHO South-East Asia Region have now eliminated
m Korea and Timor-Leste eliminated measles. India has
measles. In 2017-18 Bhutan, Maldives,aDPR
u p but has set the target year of 2020. Hence, statement 3
an
not yet achieved the measles free status
is incorrect. as
s -d
Da
http://www.searo.who.int/mediacentre/releases/2019/1712/en/

p am
u following statements regarding The Tribal Cooperative Marketing Development
82 Considernthe
A
Federation of India (TRIFED)
1. It is a national-level apex organization functioning under the administrative control of Ministry of
Tribal Affairs.
2. It provides Handloom development training for tribal population.
3. The objective of TRIFED is socio-economic development of tribal people in the country by way of
marketing development of the tribal products.

Which of the statements given above is/are correct?


A. 1 and 2 only
B. 1, 2 and 3
C. 2 and 3 only
D. 1 and 2 only

Your Answer : B
Correct Answer : B

prelims.insightsonindia.com 45
© Insights Active Learning | All rights reserved - 131815. You may not reproduce, distribute or exploit the contents in any form without
written permission by copyright owner. Copyright infringers may face civil and criminal liability
Total Marks : 200
Online Prelims TEST - 5 (SUBJECT WISE)
( InsightsIAS Mock Test Series for UPSC Preliminary Exam 2020 ) Mark Scored : 82

Answer Justification :

All the statements given above are correct.

The Tribal Cooperative Marketing Development Federation of India (TRIFED) came into existence
in 1987. It is a national-level apex organization functioning under the administrative control of
Ministry of Tribal Affairs, Govt. of India.

The ultimate objective of TRIFED is socio-economic development of tribal people in the country by
way of marketing development of the tribal products on which the lives of tribals depends heavily as
they spend most of their time and derive major portion of their income.

It provides training for Handloom and Handicraft development.

http://trifed.in/trifed/(S(jzkhqxpxxsqzodtxqnctjl1e))/Default.aspx

83 Consider the following statements


0 34
1
1. The Organisation for Economic Co-operation and Development (OECD) is an4international non-
4 6
governmental organisation that works to build better policies for better5lives.
2. Most OECD member countries are high-income economies with a-very 87 high Human Development
Index
. c om
3. Recently, India became the member of OECD il a
@ gm
2
Which of the statements given above is/are correct?
2
A. 1 and 2 only m
B. 1 only u pa
n
C. 2 and 3 only
a sa
D. 2 only
s -d
a
Your Answer :m A D
pa : D
Correct Answer
u
An
Answer Justification :

The Organisation for Economic Co-operation and Development is an intergovernmental


economic organisation with 36 member countries, founded in 1961 to stimulate economic
progress and world trade. Hence Statement 1 is incorrect.

It is a forum of countries describing themselves as committed to democracy and the market


economy, providing a platform to compare policy experiences, seek answers to common problems,
identify good practices and coordinate domestic and international policies of its members.

Most OECD members are high-income economies with a very high Human Development Index (HDI)
and are regarded as developed countries. Hence statement 2 is correct.

India is not a member of OECD. Hence Statement 3 is incorrect.

https://www.oecd.org/about/

prelims.insightsonindia.com 46
© Insights Active Learning | All rights reserved - 131815. You may not reproduce, distribute or exploit the contents in any form without
written permission by copyright owner. Copyright infringers may face civil and criminal liability
Total Marks : 200
Online Prelims TEST - 5 (SUBJECT WISE)
( InsightsIAS Mock Test Series for UPSC Preliminary Exam 2020 ) Mark Scored : 82

84 Consider the following statements


1. Mekedatu project is proposed on Cauvery River.
2. Mullaperiyar dam is located in Tamil Nadu.

Which of the statements given above is/are correct?


A. 1 only
B. 2 only
C. Both 1 and 2
D. Neither 1 nor 2

Your Answer : A
Correct Answer : A

Answer Justification :

‘Mekedatu’, literally ‘Goat’s leap’ in Kanna, is at the confluence of Cauvery and Arkavathi rivers,
4
near Kanakapura which is at a distance of 110 km from Bengaluru. 03
6 41
4
5 of Rs 5,912, was aimed at
A multi-purpose balancing reservoir project over Mekedatu, built at a cost
87 This project was also
solving the drinking water problems of Bengaluru and Ramnagar-district.
touted as one that could generate hydroelectricity to meet the
. c ompower demand in the state. Hence
Statement 1 is correct. il
g ma
Mullaiperiyar Dam is a masonry gravity dam on
2 @the Periyar River in the Indian state of Kerala It is
located 881 m above mean sea level, on the
a m2Cardamom Hills of the Western Ghats in Thekkady,
up Hence Statement 2 is incorrect.
Idukki District of Kerala, South India.
n
a sa
-d
https://www.thehindu.com/news/national/karnataka/cm-calls-on-pm-seeks-clearance-for-dpr-on-mek
s
edatu-project/article28840443.ece
Da
a m
85 Consider the p
u following statements regarding Inner Permit Line (ILP)
A n
1. It is a temporary travel document which has to be possessed by an Indian citizen to enter
‘protected’ areas of the Northeast.
2. North Eastern Council issues ILP under the Bengal Eastern Frontier Regulation, 1873.
3. ILP is applicable to all the states in North Eastern India except Tripura and Assam.

Which of the statements given above is/are correct?


A. 1 and 2 only
B. 2 and 3 only
C. 1 only
D. 1 and 3 only

Your Answer : C
Correct Answer : C

Answer Justification :

Inner line permit (ILP) is a temporary travel document which has to be possessed by an Indian
prelims.insightsonindia.com 47
© Insights Active Learning | All rights reserved - 131815. You may not reproduce, distribute or exploit the contents in any form without
written permission by copyright owner. Copyright infringers may face civil and criminal liability
Total Marks : 200
Online Prelims TEST - 5 (SUBJECT WISE)
( InsightsIAS Mock Test Series for UPSC Preliminary Exam 2020 ) Mark Scored : 82

citizen to enter ‘protected’ areas of the Northeast. Hence Statement 1 is correct.

The Central government issues the ILP under the Bengal Eastern Frontier Regulation, 1873, which
restricted the entry of ‘British subjects’ or Indians into these areas primarily to protect the British
interest in tea and oil. Hence Statement 2 is incorrect.

The restriction continued for ‘Citizens of India’ after Independence to protect tribal cultures in the
northeastern region and to regulate movement to certain areas near the international border +
Apart from the entire State of Nagaland barring its commercial hub Dimapur, the ILP is applicable
in Arunachal Pradesh and Mizoram. Hence Statement 3 is incorrect.

https://www.thehindu.com/news/national/other-states/plea-in-sc-seeks-protection-for-non-nagas-in-di
mapur/article28118917.ece

86 Which of the following is/are the necessary changes brought in Jammu and Kashmir (J&K) after the
scrapping of Article 370 and Article 35A.
0
1. People from outside J&K will be able to buy property in the state and settle there.34
1
2. Central laws will automatically apply to the state. 64 4
5
Select the correct answer using the code given below - 87
A. 1 only
. c om
B. 2 only a il
C. Both 1 and 2
@ gm
D. Neither 1 or 2 2 2
am
Your Answer : C up
Correct Answer : C s an
- da
as :
Answer Justification
D
Both the p am
statements are correct.
u
An

prelims.insightsonindia.com 48
© Insights Active Learning | All rights reserved - 131815. You may not reproduce, distribute or exploit the contents in any form without
written permission by copyright owner. Copyright infringers may face civil and criminal liability
Total Marks : 200
Online Prelims TEST - 5 (SUBJECT WISE)
( InsightsIAS Mock Test Series for UPSC Preliminary Exam 2020 ) Mark Scored : 82

4
1 03
4 64
5
- 87
m
.co
https://www.insightsonindia.com/2019/08/07/insights-into-editorial-this-is-what-has-changed-in-jam
i l
ma
mu-and-kashmir/
g
87 Consider the following pairs 2 2@
m
Navigation System
u pa
Developed/Owned by
n
1. GALILEO
a sa European Union
2. GLONASS
s -d Russia
3. QUASI-ZENITH a South Korea
D
p am
n u
A
Which of the pairs given above is/are matched correctly?
A. 2 only
B. 1 and 3 only
C. 1 and 2 only
D. 2 and 3 only

Your Answer :
Correct Answer : C

Answer Justification :

GLONASS is an acronym, which stands for Globalnaya Navigazionnaya Sputnikovaya Sistema, or


Global
Navigation Satellite System. GLONASS is Russia’s version of GPS (Global Positioning
System).

List of Global Navigation Satellite Systems:

prelims.insightsonindia.com 49
© Insights Active Learning | All rights reserved - 131815. You may not reproduce, distribute or exploit the contents in any form without
written permission by copyright owner. Copyright infringers may face civil and criminal liability
Total Marks : 200
Online Prelims TEST - 5 (SUBJECT WISE)
( InsightsIAS Mock Test Series for UPSC Preliminary Exam 2020 ) Mark Scored : 82

 GPS of the United States of America.


 Galileo of the European Union.
 IRNSS or NAVIC of India.
 Quasi-Zenith Satellite System (QZSS) of Japan

88 Consider the following statements


1. Oxytocin hormone plays an important role in reproduction, child birth and lactation.
2. Oxytocin is used to increase the size of vegetables such as pumpkins.
3. Recently, Ministry of Health and Family Welfare banned the import of Oxytocin and its
formulations.

Which of the statements given above is/are correct?


A. 1 only
B. 1 and 2 only
C. 2 and 3 only
4
D. 1, 2 and 3
1 03
4 64
Your Answer : D 5
Correct Answer : D
- 87
.com
Answer Justification :
a i l
m
@g
All the statements given above are correct.
2
m 2
a has restricted the manufacture of Oxytocin formulations
The Ministry of Health and Family Welfare
p
u
for domestic use to public sectornonly. Coupled with this, it has also banned the import of Oxytocin
a
as comes into effect from July 1.
and its formulations. The order
s -d
Da
Only Karnataka Antibiotics & Pharmaceuticals Ltd. (KAPL), a public sector company, will
m drug for domestic use. It will supply the drug to registered hospitals and clinics in
manufacture this
a
p
u private sector directly.
public and
An
The drug is widely misused in the dairy industry. Oxytocin is also used to increase the size of
vegetables such as pumpkins, watermelons, eggplants, gourds, and cucumbers.

Learning: Oxytocin has also been dubbed the hug hormone, cuddle chemical, moral molecule, and
the bliss hormone due to its effects on behaviour, including its role in love and in female
reproductive biological functions in reproduction

89 Consider the following statements with reference to the International North-South Transport
Corridor:
1. India, Iran, Azerbaijan and Russia are its founder members.
2. The corridor connects the Indian Ocean and Persian Gulf to the Caspian Sea via Iran, then connects
Northern Europe via Russia.

Which of the statements given above is/are correct?


A. 1 only

prelims.insightsonindia.com 50
© Insights Active Learning | All rights reserved - 131815. You may not reproduce, distribute or exploit the contents in any form without
written permission by copyright owner. Copyright infringers may face civil and criminal liability
Total Marks : 200
Online Prelims TEST - 5 (SUBJECT WISE)
( InsightsIAS Mock Test Series for UPSC Preliminary Exam 2020 ) Mark Scored : 82

B. 2 only
C. Both 1 and 2
D. Both 1 and 2

Your Answer : C
Correct Answer : B

Answer Justification :

International North-South Transport Corridor (INSTC), is a multi modal transportation established


in 2000 by Iran, Russia and India (Founding Members) for the purpose of promoting transportation
cooperation among the Member States. Hence Statement 1 is incorrect.

This corridor connects India Ocean and Persian Gulf to the Caspian Sea via Islamic republic of Iran,
then is connected to St. Petersburg and North European via Russian Federation. Hence Statement
2 is correct.
0 34
The INSTC has been expanded to include eleven new members, namely: Republic
6 41 of Azerbaijan,
7 54of Tajikistan, Republic of
- 8 (Observer).
Republic of Armenia, Republic of Kazakhstan, Kyrgyz Republic, Republic
Turkey, Republic of Ukraine, Republic of Belarus, Oman, Syria, Bulgaria
. c om
a il
90 Consider the following statements
@ gm
1. Turkey is the only country to share border with
2 2 both Iran and Iraq
m
pa
2. Ghouta is a city located in Iraq.
n u
Which of the statements given
a saabove is/are correct?
A. 1 only
s -d
B. 2 only
Da
m2
C. Both 1 and
a
p
u 1 nor 2
D. Neither
An
Your Answer :
Correct Answer : A

Answer Justification :

prelims.insightsonindia.com 51
© Insights Active Learning | All rights reserved - 131815. You may not reproduce, distribute or exploit the contents in any form without
written permission by copyright owner. Copyright infringers may face civil and criminal liability
Total Marks : 200
Online Prelims TEST - 5 (SUBJECT WISE)
( InsightsIAS Mock Test Series for UPSC Preliminary Exam 2020 ) Mark Scored : 82

4
1 03
4 64
5
- 87
.com
a i l
g m
2 2@
am
up
s an
- da
s
Da
m
u pa
An

Ghouta city is located in Syria.

91 Consider the following statements regarding World Heritage Committee


1. The World Heritage Committee meets once a year, and consists of representatives from 21 of the

prelims.insightsonindia.com 52
© Insights Active Learning | All rights reserved - 131815. You may not reproduce, distribute or exploit the contents in any form without
written permission by copyright owner. Copyright infringers may face civil and criminal liability
Total Marks : 200
Online Prelims TEST - 5 (SUBJECT WISE)
( InsightsIAS Mock Test Series for UPSC Preliminary Exam 2020 ) Mark Scored : 82

States Parties to the Convention elected by their General Assembly.


2. The World Heritage Committee meets once a year, and consists of representatives from 21 of the
States Parties to the Convention elected by their General Assembly.
3. It has the final say on whether a property is inscribed on the World Heritage List.

Which of the statements given above is/are correct?


A. 1 and 2 only
B. 2 and 3 only
C. 1, 2 and 3
D. 1 and 3 only

Your Answer :
Correct Answer : D

Answer Justification :

4
All the statements given above are correct.
1 03
4 64
75its first session, the
The World Heritage Committee meets once a year, and consists of representatives from 21 of the
States Parties to the Convention elected by their General Assembly.8At
-
m Committee.
Committee adopted its Rules of Procedure of the World Heritage
i l .co
The Committee is responsible for the implementationaof the World Heritage Convention, defines the
@ gm assistance upon requests from States
use of the World Heritage Fund and allocates financial
2 2 is inscribed on the World Heritage List.
Parties. It has the final say on whether a property
pam
It examines reports on the state n u
of conservation of inscribed properties and asks States Parties to
s a
take action when propertiesaare not being properly managed. It also decides on the inscription or
d
deletion of properties on- the List of World Heritage in Danger.
s
Da
am
https://whc.unesco.org/en/committee/
p
u
An
92 Consider the following statements regarding International Court of Justice
1. It was established in June 1920 by the Charter of the United Nations and began work in April 1921.
2. The seat of the Court is at the Peace Palace in The Hague
3. It has no jurisdiction to deal with applications from individuals, NGOs or private groups

Which of the statements given above is/are correct?


A. 1 and 3 only
B. 2 and 3 only
C. 2 only
D. 1 and 2 only

Your Answer : B
Correct Answer : B

Answer Justification :

prelims.insightsonindia.com 53
© Insights Active Learning | All rights reserved - 131815. You may not reproduce, distribute or exploit the contents in any form without
written permission by copyright owner. Copyright infringers may face civil and criminal liability
Total Marks : 200
Online Prelims TEST - 5 (SUBJECT WISE)
( InsightsIAS Mock Test Series for UPSC Preliminary Exam 2020 ) Mark Scored : 82

The International Court of Justice (ICJ) is the principal judicial organ of the United Nations (UN). It
was established in June 1945 by the Charter of the United Nations and began work in April 1946.
Hence Statement 1 is incorrect.

The seat of the Court is at the Peace Palace in The Hague (Netherlands). Of the six principal organs
of the United Nations, it is the only one not located in New York (United States of America). Hence
Statement 2 is correct.

The Court’s role is to settle, in accordance with international law, legal disputes submitted to it by
States and to give advisory opinions on legal questions referred to it by authorized United Nations
organs and specialized agencies.

It has no jurisdiction to deal with applications from individuals, NGOs or private groups and it rules
only on the rights and obligations of States. Hence Statement 3 is correct.

https://www.icj-cij.org/en/court
4
1 03
64
93 Which of the following wildlife sanctuaries and National Parks are located in Assam?
4
1. Nameri National Park
8 75
2. Kaziranga National Park -
m
.co
3. Gorumara National Park
4. Pobitora wildlife Sanctuary i l
g ma
2@below
Select the correct answer using the code given
2
A. 1 and 2 only m
B. 2 only u pa
n
C. 1, 3 and 4 only
a sa
D. 1, 2 and 4 only - d

D as
Your Answer :m D
Correct u pa : D
Answer
An
Answer Justification :

Nameri National Park, Kaziranga National Park and Pobitora wildlife Sanctuary located in Assam.
Gorumara National Park is located in West Bengal.

https://www.thehindu.com/sci-tech/energy-and-environment/assam-floods-why-we-need-to-act-fast-t
o-save-kaziranga-and-its-wildlife/article28720848.ece

94 Consider the following statements regarding ‘Drought Management in India’


1. There is no single, legally accepted definition of drought in India.
2. Drought will fall within the definition of “disaster” as defined in Disaster Management Act, 2005.
3. State Government is the final authority when it comes to declaring a region as drought affected

Which of the statements given above is/are correct?


A. 3 only

prelims.insightsonindia.com 54
© Insights Active Learning | All rights reserved - 131815. You may not reproduce, distribute or exploit the contents in any form without
written permission by copyright owner. Copyright infringers may face civil and criminal liability
Total Marks : 200
Online Prelims TEST - 5 (SUBJECT WISE)
( InsightsIAS Mock Test Series for UPSC Preliminary Exam 2020 ) Mark Scored : 82

B. 2 and 3 only
C. 1 and 2 only
D. 1, 2 and 3

Your Answer : D
Correct Answer : D

Answer Justification :

All the statements given above are correct.

Please refer below link for overall understanding of the drought issue in India.

http://www.arthapedia.in/index.php?title=Drought

95 ‘Fiscal Monitor Report’ is prepared by 4


1 03
A. European Central Bank
4 64
5
B.
C.
International Monetary Fund
International Bank for Reconstruction and Development - 87
m
D. co
Organization for Economic Cooperation and Development
il .
m a
Your Answer : B
@ g
Correct Answer : B
m 22
u pa
Answer Justification :
a n
s
Key Publications of IMF- da
s
Publicationsm
Da
a
nup
A
1. World economic outlook

2. Global financial stability report

3. Fiscal monitor

4. Regional economic prospects

5. Finance and Development

96 Consider the following statements


1. Codex Alimentarius Commission (CAC) is an intergovernmental body with more than 180 members.
2. It was jointly established by the Food and Agriculture Organization of the United Nations (FAO) and

prelims.insightsonindia.com 55
© Insights Active Learning | All rights reserved - 131815. You may not reproduce, distribute or exploit the contents in any form without
written permission by copyright owner. Copyright infringers may face civil and criminal liability
Total Marks : 200
Online Prelims TEST - 5 (SUBJECT WISE)
( InsightsIAS Mock Test Series for UPSC Preliminary Exam 2020 ) Mark Scored : 82

the World Health Organization (WHO) in 1990’s.


3. Codex Alimentarius is recognized by the World Trade Organization as an international reference
point for the resolution of disputes concerning food safety and consumer protection.

Which of the statements given above is/are correct?


A. 3 only
B. 2 and 3 only
C. 1, 2 and 3
D. 1 and 3 only

Your Answer :
Correct Answer : D

Answer Justification :

The Codex Alimentarius Commission (CAC) is an intergovernmental body with over 180
0
members, within the framework of the Joint Food Standards Programme established34 by the Food
41 Organization
and Agriculture Organization of the United Nations (FAO) and the World Health
6
54 fair practices in the
(WHO), with the purpose of protecting the health of consumers and ensuring
7
food trade. Hence Statement 1 is correct. -8
om
l.c
It was established in 1960’s. Hence Statement 2 is incorrect.
a i
m
The Codex Alimentarius is recognized by the
2 @gWorld Trade Organization as an international
2 concerning food safety and consumer protection.
reference point for the resolution of disputes
m
Hence Statement 3 is correct. pa
n u
a sa
s -d
97 Consider the following statements regarding Global Carbon Project
1. It was establishedDina2001 by a shared partnership between the International Geosphere-Biosphere
am the International Human Dimensions Programme on Global Environmental
Programme (IGBP),
p
Change u (IHDP) and others.
A n
2. It was formed to work with the international science community to establish a common and
mutually agreed knowledge base to support policy debate and action to slow down and ultimately
stop the increase of greenhouse gases in the atmosphere.

Which of the statements given above is/are correct?


A. 1 only
B. 2 only
C. Both 1 and 2
D. Neither 1 nor 2

Your Answer :
Correct Answer : C

Answer Justification :

Both the statements are correct.

prelims.insightsonindia.com 56
© Insights Active Learning | All rights reserved - 131815. You may not reproduce, distribute or exploit the contents in any form without
written permission by copyright owner. Copyright infringers may face civil and criminal liability
Total Marks : 200
Online Prelims TEST - 5 (SUBJECT WISE)
( InsightsIAS Mock Test Series for UPSC Preliminary Exam 2020 ) Mark Scored : 82

The Global Carbon Project was established in 2001 by a shared partnership between the
International Geosphere-Biosphere Programme (IGBP), the International Human
Dimensions Programme on Global Environmental Change (IHDP), the World Climate
Research Programme (WCRP) and Diversitas. This partnership constituted the Earth Systems
Science Partnership (ESSP) which subsequently evolved into Future Earth.

It was formed to work with the international science community to establish a common and
mutually agreed knowledge base to support policy debate and action to slow down and ultimately
stop the increase of greenhouse gases in the atmosphere.

The overwhelming realization that anthropogenic climate change is a reality has focused the
attention of the scientific community, policymakers and the general public on the rising
atmospheric concentrations of the main greenhouse gases, carbon dioxide (CO2), methane (CH4),
and nitrous oxide (N2O).

The GCP has approached this challenge by focusing comprehensively on the global biogeochemical
34 drivers, and
cycles which govern these three greenhouse gases, including their natural and human
0
opportunities for low carbon pathways. 1 4
6
https://www.globalcarbonproject.org/about/index.htm 7 54
-8
. c om
98 Consider the following statements regarding ‘Zero Budget
a il Natural Farming’ (ZBNF)
1. Zero Budget Natural Farming (ZBNF) is a farming gm
@any
practice that believes in natural growth of crops
without adding any fertilizers and pesticides2or other foreign elements.
2. The word Zero Budget refers to the zerom
2
net cost of production of all crops.
pa
nu
3. The father of ZBNF is MS Swaminathan.
s a
da above is/are correct?
Which of the statements- given
A. 1 only
D as
B. 1 and 2 m
only
2p
C. 1, u
a
and 3
n
D.ANone

Your Answer : B
Correct Answer : B

Answer Justification :

Zero-Budget Natural Farming (ZBNF) is a holistic alternative to the present paradigm of high-cost
chemical inputs-based agriculture. It is very effective in addressing the uncertainties of climate
change.

ZBNF principles are in harmony with the principles of Agroecology. Its uniqueness is that it is based
on the latest scientific discoveries in Agriculture, and, at the same time it is rooted in Indian
tradition.

UN-FAO in April 2018 urged all countries to move towards the adoption of Agroecology to meet the
twin goals of global food security and conservation of the environment.

prelims.insightsonindia.com 57
© Insights Active Learning | All rights reserved - 131815. You may not reproduce, distribute or exploit the contents in any form without
written permission by copyright owner. Copyright infringers may face civil and criminal liability
Total Marks : 200
Online Prelims TEST - 5 (SUBJECT WISE)
( InsightsIAS Mock Test Series for UPSC Preliminary Exam 2020 ) Mark Scored : 82

The father of ZBNF is Subhash Palekar. Hence Statement 3 is incorrect.

http://apzbnf.in/

99 Consider the following statements regarding All India Tiger Estimation - 2018
1. Madhya Pradesh state has the highest number of tigers in India followed by Karnataka
2. India conducts the All India Tiger Estimation every five years.
3. Pench sanctuary and Periyar sanctuary emerged as the best managed tiger reserves in the country.

Which of the statements given above is/are correct?


A. 1 and 2 only
B. 1 and 2 only
C. 3 only
D. 1 and 3 only

4
03
Your Answer : D
Correct Answer : D 1
4 64
5
Answer Justification :
- 87
o m
The Tiger Estimation exercise is believed to be the world’s
i l .clargest wildlife survey effort in terms of

ma
coverage, intensity of sampling and quantum of camera trapping.
g
@ four years. Three cycles of the estimation have
2
India conducts the All India Tiger Estimation2every
already been completed in 2006, 2010aandm 2014. Hence Statement 2 is incorrect.
u p
n
Madhya Pradesh saw the highest
a sa number at 526, closely followed by Karnataka (524) and
Uttarakhand (442). Hence
s - dStatement 1 is correct.
a
Madhya Pradesh'sDPench sanctuary and Kerala’s Periyar sanctuary emerged as the best managed
p am
u in the country. Hence Statement 3 is correct.
tiger reserves
n
A
http://pib.nic.in/newsite/PrintRelease.aspx?relid=192351

100 Consider the following statements regarding ‘Hope Spots’.


1. Andaman and Nicobar Islands and Lakshadweep islands are the first in India to make it to the list of
global hope spots.
2. A hope spot is an area of an ocean that needs special protection because of its wildlife and
significant underwater habitats.
3. They are declared by Intergovernmental Oceanographic Commission (IOC).

Which of the statements given above is/are correct?


A. 1 and 3 only
B. 2 and 3 only
C. 1 and 2 only
D. 1, 2 and 3 only

prelims.insightsonindia.com 58
© Insights Active Learning | All rights reserved - 131815. You may not reproduce, distribute or exploit the contents in any form without
written permission by copyright owner. Copyright infringers may face civil and criminal liability
Total Marks : 200
Online Prelims TEST - 5 (SUBJECT WISE)
( InsightsIAS Mock Test Series for UPSC Preliminary Exam 2020 ) Mark Scored : 82

Your Answer : D
Correct Answer : C

Answer Justification :

Andaman and Nicobar Islands and Lakshadweep islands have recently been named as the new
"hope spots" by the International Union for Conservation of Nature (IUCN) and Mission Blue, an
organization involved in the study of oceans. Hence Statement 1 is correct and Statement 3 is
incorrect.

A hope spot is an area of an ocean that needs special protection because of its wildlife and
significant underwater habitats. Hence Statement 2 is correct.

The two group of islands, considered extremely rich in marine biodiversity, are the first places in
India to have been added in the list of 50 global 'hope spots'. Earlier, 13 hope spots had been
identified all around the world. The additional list of 31 new hope spots was released by IUCN and
4
03
Mission Blue on October 23.
1
4 64
Some of these areas are already designated as protected areas by the government while the others
need special protection. 7 5
-8
. c om
a il
@ gm
2 2
m
u pa
n
a sa
s -d
Da
p am
u
An

prelims.insightsonindia.com 59
© Insights Active Learning | All rights reserved - 131815. You may not reproduce, distribute or exploit the contents in any form without
written permission by copyright owner. Copyright infringers may face civil and criminal liability

You might also like